40

Click here to load reader

Compression Guide Two-Dimensional Motion and Vectorsaeascience.weebly.com/uploads/3/1/9/8/31980129/honors_vectors.pdf · Section 1 Introduction to Vectors ... SE Sample Set D Projectiles

Embed Size (px)

Citation preview

Page 1: Compression Guide Two-Dimensional Motion and Vectorsaeascience.weebly.com/uploads/3/1/9/8/31980129/honors_vectors.pdf · Section 1 Introduction to Vectors ... SE Sample Set D Projectiles

CHAPTER REVIEW, ASSESSMENT, AND STANDARDIZED TEST PREPARATION

Online and Technology Resources

Visit go.hrw.com to find a variety of online resources. To access this chapter’s extensions, enter the keyword HF6TDMXT and click the “go” button. Click Holt Online Learning for an online edition of this textbook, and other interactive resources.

Planning Guide

Chapter Openerpp. 80 – 81 CD Visual Concepts, Chapter 3 b ANC Discovery Lab Vector Treasure Hunt*◆ b

Section 1 Introduction to Vectors• Distinguish between a scalar and a vector.• Add and subtract vectors by using the graphical method.• Multiply and divide vectors by scalars.

OSP Lesson Plans TR 6 Graphical Method of Vector Addition TR 14A Commutative Property of Vectors

TE Demonstration Vector Addition, p. 83 g

OSP Lesson Plans TR 15A The Pythagorean Theorem and the

Tangent Function TR 16A Adding Vectors That Are Not

Perpendicular TR 17A Adding Vectors Algebraically

80A Chapter 3 Two-Dimensional Motion and Vectors

OBJECTIVES LABS, DEMONSTRATIONS, AND ACTIVITIES TECHNOLOGY RESOURCES

Two-Dimensional Motion and Vectors To shorten instruction because of time limitations, omit the opener and Section 4 and abbreviate the review.

Compression Guide

CHAPTER 3

pp. 86 – 94 PACING • 45 min

PACING • 45 min

SE Chapter Highlights, p. 107 SE Chapter Review, pp. 108 –113 SE Graphing Calculator Practice, p. 113 g SE Alternative Assessment, p. 113 a SE Standardized Test Prep, pp. 114 –115 g SE Appendix D: Equations, pp. 854 – 855 SE Appendix I: Additional Problems, pp. 881– 882ANC Study Guide Worksheet Mixed Review* gANC Chapter Test A* gANC Chapter Test B* a OSP Test Generator

PACING • 90 min

Section 2 Vector Operations• Identify appropriate coordinate systems for solving problems

with vectors.• Apply the Pythagorean theorem and tangent function to cal-

culate the magnitude and direction of a resultant vector.• Resolve vectors into components using the sine and cosine

functions.• Add vectors that are not perpendicular.

OSP Lesson Plans TR 7 Vertical Motion of a Projectile

TE Demonstration Air Resistance, p. 96 g SE Quick Lab Projectile Motion, p. 97 g SE Inquiry Lab Velocity of a Projectile, pp. 116 –117◆

ANC Datasheet Inquiry Lab, Velocity of a Projectile* g ANC Datasheet Skills Practice Lab, Velocity of a Projectile* ANC Invention Lab The Path of a Human Cannonball*◆

ANC CBLTM Experiment Projectile Motion*◆ a

pp. 95 –101

Section 3 Projectile Motion• Recognize examples of projectile motion.• Describe the path of a projectile as a parabola. • Resolve vectors into their components and apply the kine-

matic equations to solve problems involving projectile motion.

PACING • 135 min

Section 4 Relative Motion• Describe situations in terms of frame of reference.• Solve problems involving relative velocity.

pp. 102–105 Advanced Level OSP Lesson Plans TR 8 Frames of Reference

PACING • 45 min

pp. 82 – 85PACING • 45 min

• Holt Calendar Planner• Customizable Lesson Plans• Editable Worksheets• ExamView® Version 6

Assessment Suite

• Interactive Teacher’s Edition• Holt PuzzlePro®

• Holt PowerPoint® Resources

• MindPoint® Quiz Show

This DVD package includes:

Page 2: Compression Guide Two-Dimensional Motion and Vectorsaeascience.weebly.com/uploads/3/1/9/8/31980129/honors_vectors.pdf · Section 1 Introduction to Vectors ... SE Sample Set D Projectiles

www.scilinks.orgMaintained by the National Science Teachers Association.

Topic: VectorsSciLinks Code: HF61597

Topic: Projectile MotionSciLinks Code: HF61223

SE Section Review, p. 85 g ANC Study Guide Worksheet Section 1* g ANC Quiz Section 1* b

SE Sample Set A Finding Resultant Magnitude, pp. 88 – 89 g TE Classroom Practice, p. 88 g ANC Problem Workbook* and OSP Problem Bank Sample Set A g SE Sample Set B Resolving Vectors, pp. 91– 92 g TE Classroom Practice, p. 91 g ANC Problem Workbook* and OSP Problem Bank Sample Set B g SE Sample Set C Adding Vectors Algebraically, pp. 93 – 94 g TE Classroom Practice, p. 93 g ANC Problem Workbook* and OSP Problem Bank Sample Set C g

SE Section Review, p. 94 g ANC Study Guide Worksheet Section 2* g ANC Quiz Section 2* b

Chapter 3 Planning Guide 80B

SE Sample Set D Projectiles Launched Horizontally, pp. 98 – 99 g TE Classroom Practice, p. 98 g ANC Problem Workbook* and OSP Problem Bank Sample Set D g SE Sample Set E Projectiles Launched at an Angle, pp. 100 –101 g TE Classroom Practice, p. 100 g ANC Problem Workbook* and OSP Problem Bank Sample Set E g

SE Section Review, p. 101 g ANC Study Guide Worksheet Section 3* g ANC Quiz Section 3* b

SKILLS DEVELOPMENT RESOURCES REVIEW AND ASSESSMENT CORRELATIONS

SE Section Review, p. 105 a ANC Study Guide Worksheet Section 4* g ANC Quiz Section 4* g

SE Conceptual Challenge, p. 103 a SE Sample Set F Relative Velocity, pp. 104 –105 g TE Classroom Practice, p. 104 g ANC Problem Workbook* and OSP Problem Bank Sample Set F g SE Appendix J: Advanced Topics Special Relativity and Velocities,

pp. 916 – 917 a

KEY SE Student Edition TE Teacher Edition ANC Ancillary Worksheet

OSP One-Stop Planner CD CD or CD-ROM TR Teaching Transparencies

EXT Online Extension * Also on One-Stop Planner ◆ Requires advance prep

• Guided Reading Audio Program• Student One Stop• Virtual Investigations• Visual Concepts

Search for any lab by topic, standard, difficulty level, or time. Edit any lab to fit your needs, or create your own labs. Use the Lab Materials QuickList software to customize your lab materials list.

ClassroomCD-ROMs

National ScienceEducation Standards

UCP 2, 3SAI 1, 2

UCP 2, 3

UCP 2, 3SAI 1, 2

UCP 2, 3

Page 3: Compression Guide Two-Dimensional Motion and Vectorsaeascience.weebly.com/uploads/3/1/9/8/31980129/honors_vectors.pdf · Section 1 Introduction to Vectors ... SE Sample Set D Projectiles

CHAPTER XCHAPTER 3Overview

80

Section 1 discusses scalar andvector quantities and graphicalvector addition.

Section 2 explains the use of thePythagorean theorem andtrigonometric functions to findresultant vectors and vectorcomponents.

Section 3 explores projectilemotion, neglecting air resistance.

Section 4 describes relativemotion in terms of vector operations.

About the IllustrationThis fountain is located inBayfront Park in Sarasota, Florida.Point out that each drop of wateremerging from the fountainmoves as a projectile and obeys therules for projectile motiondescribed in this chapter. Likewise,the dolphins move as projectiles asthey sail through the air.

Page 4: Compression Guide Two-Dimensional Motion and Vectorsaeascience.weebly.com/uploads/3/1/9/8/31980129/honors_vectors.pdf · Section 1 Introduction to Vectors ... SE Sample Set D Projectiles

8181

Without air resistance, any object that is thrown orlaunched into the air and that is subject to gravitationalforce will follow a parabolic path. The water droplets in thisfountain are one example. The velocity of any object in two-dimensional motion—such as one of these waterdroplets—can be separated into horizontal and verticalcomponents, as shown in the diagram.

CHAPTER 3

vy

vx

vy

vx

vy

vx

vy

vx

vy

vx

vx

Two-DimensionalMotion andVectors

WHAT TO EXPECTIn this chapter, you will use vectors to analyzetwo-dimensional motion and to solve problemsin which objects are projected into the air.

CHAPTER PREVIEW

1 Introduction to VectorsScalars and VectorsProperties of Vectors

2 Vector OperationsCoordinate Systems in Two DimensionsDetermining Resultant Magnitude and

DirectionResolving Vectors into ComponentsAdding Vectors That Are Not

Perpendicular3 Projectile Motion

Two-Dimensional Motion4 Relative Motion

Frames of ReferenceRelative Velocity

Knowledge to Review✔ Displacement is a change in

location relative to a refer-ence point.

✔ Velocity includes speed anddirection.

✔ Acceleration is the rate ofchange of velocity withrespect to time. It has bothmagnitude and direction.

Items to Probe✔ Displacement versus dis-

tance: Have students decidedisplacement and distancevalues for scenarios listed onthe board, such as roundtrips versus one-way trips,or by walking from oneplace to another in the classroom.

✔ Acceleration and velocity:Ask students to describe theeffects of the four possiblecombinations of accelerationand velocity in one dimen-sion: a > 0, v > 0; a < 0, v > 0;a > 0, v < 0; a < 0, v < 0.

Tapping PriorKnowledge

Why

After you know how to analyze two-dimensionalmotion, you can predict where a falling objectwill land based on its initial velocity and position.

Why It Matters

Page 5: Compression Guide Two-Dimensional Motion and Vectorsaeascience.weebly.com/uploads/3/1/9/8/31980129/honors_vectors.pdf · Section 1 Introduction to Vectors ... SE Sample Set D Projectiles

82

Introduction to VectorsSECTION 1General Level SECTION 1

SCALARS AND VECTORS

In the chapter “Motion in One Dimension,” our discussion of motion was lim-

ited to two directions, forward and backward. Mathematically, we described

these directions of motion with a positive or negative sign. That method

works only for motion in a straight line. This chapter explains a method of

describing the motion of objects that do not travel along a straight line.

Vectors indicate direction; scalars do not

Each of the physical quantities encountered in this book can be categorized as

either a scalar quantity or a vector quantity. A is a quantity that has

magnitude but no direction. Examples of scalar quantities are speed, volume,

and the number of pages in this textbook. A is a physical quantity that

has both direction and magnitude.

As we look back to the chapter “Motion in One Dimension,” we can see that

displacement is an example of a vector quantity. An airline pilot planning a

trip must know exactly how far and which way to fly. Velocity is also a vector

quantity. If we wish to describe the velocity of a bird, we must specify both its

speed (say, 3.5 m/s) and the direction in which the bird is flying (say, north-

east). Another example of a vector quantity is acceleration.

Vectors are represented by boldface symbols

In physics, quantities are often represented by symbols, such as t for

time. To help you keep track of which symbols represent vector quan-

tities and which are used to indicate scalar quantities, this book will

use boldface type to indicate vector quantities. Scalar quantities will

be in italics. For example, the speed of a bird is written as v = 3.5 m/s.

But a velocity, which includes a direction, is written as v = 3.5 m/s to

the northeast. When writing a vector on your paper, you can distin-

guish it from a scalar by drawing an arrow above the abbreviation for

a quantity, such as v�� = 3.5 m/s to the northeast.

One way to keep track of vectors and their directions is to use

diagrams. In diagrams, vectors are shown as arrows that point in the

direction of the vector. The length of a vector arrow in a diagram is

proportional to the vector’s magnitude. For example, in Figure 1the arrows represent the velocities of the two soccer players running

toward the soccer ball.

vector

scalar

Chapter 382

SECTION OBJECTIVES

■ Distinguish between a scalarand a vector.

■ Add and subtract vectors byusing the graphical method.

■ Multiply and divide vectorsby scalars.

Teaching TipCarry an arrow with you to class,and use it often to describe thedirection of the vector.

The Language of PhysicsEstablish a convention for distin-guishing vectors from scalars foruse in your classroom. Using anarrow above the variable is a goodsubstitute for bold type, whichwould be hard to reproduce on achalkboard. Make sure to be con-sistent when using this conven-tion. For instance, when you usean arrow above the symbol to des-ignate a vector, be sure to signifythe direction of the vector as wellas its magnitude and units.

Visual Strategy

Figure 1Tell students that the arrows inthe figure represent velocity, thatis, both speed and direction.

Why do we consider velocityinstead of speed in order to

decide which player reaches theball first?

Direction is importantbecause a player may run

very fast but not toward the ball.A

Q

Figure 1The lengths of the vector arrows represent the magnitudes of these two soccer players’ velocities.

scalar

a physical quantity that has magnitude but no direction

vector

a physical quantity that has bothmagnitude and direction

Page 6: Compression Guide Two-Dimensional Motion and Vectorsaeascience.weebly.com/uploads/3/1/9/8/31980129/honors_vectors.pdf · Section 1 Introduction to Vectors ... SE Sample Set D Projectiles

83

SECTION 1A resultant vector represents the sum of two or more vectors

When adding vectors, you must make certain that they have the same units

and describe similar quantities. For example, it would be meaningless to add a

velocity vector to a displacement vector because they describe different physi-

cal quantities. Similarly, it would be meaningless, as well as incorrect, to add

two displacement vectors that are not expressed in the same units. For exam-

ple, you cannot add meters and feet together.

Section 1 of the chapter “Motion in One Dimension” covered vector addi-

tion and subtraction in one dimension. Think back to the example of the

gecko that ran up a tree from a 20 cm marker to an 80 cm marker. Then the

gecko reversed direction and ran back to the 50 cm marker. Because the two

parts of this displacement are opposite, they can be added together to give a

total displacement of 30 cm. The answer found by adding two vectors in this

way is called the

Vectors can be added graphically

Consider a student walking 1600 m to a friend’s house and then 1600 m to

school, as shown in Figure 2. The student’s total displacement during his walk

to school is in a direction from his house to the school, as shown by the dotted

line. This direct path is the vector sum of the student’s displacement from his

house to his friend’s house and his displacement from the friend’s house to

school. How can this resultant displacement be found?

One way to find the magnitude and direction of the student’s total dis-

placement is to draw the situation to scale on paper. Use a reasonable scale,

such as 50 m on land equals 1 cm on paper. First draw

the vector representing the student’s displacement

from his house to his friend’s house, giving the proper

direction and scaled magnitude. Then draw the vector

representing his walk to the school, starting with the

tail at the head of the first vector. Again give its scaled

magnitude and the right direction. The magnitude of

the resultant vector can then be determined by using a

ruler. Measure the length of the vector pointing from

the tail of the first vector to the head of the second vec-

tor. The length of that vector can then be multiplied by

50 (or whatever scale you have chosen) to get the actu-

al magnitude of the student’s total displacement in

meters.

The direction of the resultant vector may be deter-

mined by using a protractor to measure the angle

between the resultant and the first vector or between the

resultant and any chosen reference line.

resultant.

83Two-Dimensional Motion and Vectors

(b)

(a)

(c)

Figure 2A student walks from his house to his friend’s house (a), thenfrom his friend’s house to the school (b). The student’s resultantdisplacement (c) can be found by using a ruler and a protractor.

resultant

a vector that represents the sumof two or more vectors

The word vector is also used by air-line pilots and navigators. In thiscontext, a vector is the particularpath followed or to be followed,given as a compass heading.

Did you know?

Demonstration

Vector AdditionPurpose Preview force as a vector quantity to demonstratevector addition.Materials large, empty boxProcedure Ask for a student vol-unteer. Explain to the class thatyou will push the box in onedirection while the student willpush the box in a second, per-pendicular direction at the sametime. Ask students to predict themotion of the box.

Have the student volunteertake a practice push alone, andemphasize that the box moves inthe direction of the push. Take apractice push by yourself on thebox, and again emphasize thatthe box moves in the direction ofthe push. Return the box to itsoriginal location, and both youand the student push the boxperpendicular to each other atthe count of three. Ask a studentto explain why the box movedalong the diagonal. (The boxmoved in a direction between thedirections of the two pushes.)

Show the vector addition forthe demonstration qualitativelyon the chalkboard. This shouldlay the groundwork for a discus-sion of why vector addition is avaluable activity.

GENERAL

Page 7: Compression Guide Two-Dimensional Motion and Vectorsaeascience.weebly.com/uploads/3/1/9/8/31980129/honors_vectors.pdf · Section 1 Introduction to Vectors ... SE Sample Set D Projectiles

84

SECTION 1PROPERTIES OF VECTORS

Now consider a case in which two or more vectors act at the same point.

When this occurs, it is possible to find a resultant vector that has the same net

effect as the combination of the individual vectors. Imagine looking down

from the second level of an airport at a toy car moving at 0.80 m/s across a

walkway that moves at 1.5 m/s. How can you determine what the car’s resul-

tant velocity will look like from your view point?

Vectors can be moved parallel to themselves in a diagram

Note that the car’s resultant velocity while moving from one side of the walk-

way to the other will be the combination of two independent motions. Thus,

the moving car can be thought of as traveling first at 0.80 m/s across the walk-

way and then at 1.5 m/s down the walkway. In this way, we can draw a given

vector anywhere in the diagram as long as the vector is parallel to its previous

alignment (so that it still points in the same direction). Thus, you can draw

one vector with its tail starting at the tip of the other as long as the size and

direction of each vector do not change. This process is illustrated in Figure 3.Although both vectors act on the car at the same point, the horizontal vector

has been moved up so that its tail begins at the tip of the vertical vector.

The resultant vector can then be drawn from the tail of the first vector to the

tip of the last vector. This method is known as the triangle (or polygon)

method of addition.

Again, the magnitude of the resultant vector can be measured using a ruler,

and the angle can be measured with a protractor. In the next section, we will

develop a technique for adding vectors that is less time-consuming because it

involves a calculator instead of a ruler and protractor.

Vectors can be added in any order

When two or more vectors are added, the sum is independent of the order of

the addition. This idea is demonstrated by a runner practicing for a marathon

along city streets, as represented in Figure 4. The runner executes the same

four displacements in each case, but the order is different. Regardless of which

path the runner takes, the runner will have the same total displacement,

expressed as d. Similarly, the vector sum of two or more vectors is the same

regardless of the order in which the vectors are added, provided that the mag-

nitude and direction of each vector remain the same.

To subtract a vector, add its opposite

Vector subtraction makes use of the definition of the negative of a vector. The

negative of a vector is defined as a vector with the same magnitude as the origi-

nal vector but opposite in direction. For instance, the negative of the velocity

of a car traveling 30 m/s to the west is −30 m/s to the west, or +30 m/s to the

east. Thus, adding a vector to its negative vector gives zero.

MisconceptionAlert

Students may have difficulty visu-alizing the movement of the toycar on the walkway. Illustrate thesituation on the board, andemphasize that the motion repre-sented in Figure 3 is drawn asviewed by a stationary observerabove the walkway. Frame of refer-ence will be covered in Section 4.

The Language of PhysicsThe triangle method of vectoraddition is also called the polygonmethod, the head-to-tail method,or the tip-to-tail method.

Teaching TipStudents will better understandthe commutative property ofvector addition if they work sev-eral examples on paper with aruler and protractor.

Visual Strategy

Figure 4Reinforce students’ understand-ing of the difference between dis-tance and displacement bypointing out that if the runnerstarts and ends at the positionsshown in Figure 4, the runner’sdisplacement will be the sameregardless of the path the runnerchooses to follow.

How can the runner increasedistance but begin and end at

the same points?

The runner can take a differ-ent, longer path but will have

to double back somewhere alongthe path to end up at the samepoint.

A

Q

GENERAL

STOP

Chapter 384

Car

(c)

(a)(b)

vwalkway = 1.5 m/s

vresultant

v car

= 0

.80

m/s

Figure 3The resultant velocity (a) of a toycar moving at a velocity of 0.80 m/s(b) across a moving walkway with avelocity of 1.5 m/s (c) can be foundusing a ruler and a protractor.

Figure 4A marathon runner’s displacement,d, will be the same regardless of whether the runner takes path(a) or (b) because the vectors canbe added in any order.

(a)

d

(b)

d

Page 8: Compression Guide Two-Dimensional Motion and Vectorsaeascience.weebly.com/uploads/3/1/9/8/31980129/honors_vectors.pdf · Section 1 Introduction to Vectors ... SE Sample Set D Projectiles

85

SECTION 1When subtracting vectors in two dimensions, first draw the negative of the

vector to be subtracted. Then add that negative vector to the other vector by

using the triangle method of addition.

Multiplying or dividing vectors by scalars results in vectors

There are mathematical operations in which vectors can multiply other vec-

tors, but they are not needed in this book. This book does, however, make use

of vectors multiplied by scalars, with a vector as the result. For example, if a

cab driver obeys a customer who tells him to go twice as fast, that cab’s origi-

nal velocity vector, vcab, is multiplied by the scalar number 2. The result, writ-

ten 2vcab, is a vector with a magnitude twice that of the original vector and

pointing in the same direction.

On the other hand, if another cab driver is told to go twice as fast in the

opposite direction, this is the same as multiplying by the scalar number −2.

The result is a vector with a magnitude two times the initial velocity but

pointing in the opposite direction, written as −2vcab.

Teaching TipSome students may need furtherexplanation and visual examplesof the negative of a vector. Giveseveral examples of vectors indifferent directions, and thenshow their negative vectors.

85Two-Dimensional Motion and Vectors

SECTION REVIEW

1. Which of the following quantities are scalars, and which are vectors?

a. the acceleration of a plane as it takes off

b. the number of passengers on the plane

c. the duration of the flight

d. the displacement of the flight

e. the amount of fuel required for the flight

2. A roller coaster moves 85 m horizontally, then travels 45 m at an angle of

30.0° above the horizontal. What is its displacement from its starting

point? Use graphical techniques.

3. A novice pilot sets a plane’s controls, thinking the plane will fly at

2.50 × 102 km/h to the north. If the wind blows at 75 km/h toward the

southeast, what is the plane’s resultant velocity? Use graphical techniques.

4. While flying over the Grand Canyon, the pilot slows the plane down to

one-half the velocity in item 3. If the wind’s velocity is still 75 km/h

toward the southeast, what will the plane’s new resultant velocity be?

Use graphical techniques.

5. Critical Thinking The water used in many fountains is recycled.

For instance, a single water particle in a fountain travels through an 85 m

system and then returns to the same point. What is the displacement of

this water particle during one cycle?

1. a. vectorb. scalarc. scalard. vectore. scalar

2. 126 m at (1.0 × 101)° abovethe horizontal

3. 204 km/h at 75° north of east

4. 89 km/h at 54° north of east

5. zero

SECTION REVIEWANSWERS

www.scilinks.orgTopic: VectorsCode: HF61597

Page 9: Compression Guide Two-Dimensional Motion and Vectorsaeascience.weebly.com/uploads/3/1/9/8/31980129/honors_vectors.pdf · Section 1 Introduction to Vectors ... SE Sample Set D Projectiles

86

Vector OperationsSECTION 2General Level SECTION 2

COORDINATE SYSTEMS IN TWO DIMENSIONS

In the chapter “Motion in One Dimension,” the motion of a gecko climbing a

tree was described as motion along the y-axis. The direction of the displacement

of the gecko was denoted by a positive or negative sign. The displacement of the

gecko can now be described by an arrow pointing along the y-axis, as shown in

Figure 5. A more versatile system for diagraming the motion of an object, how-

ever, employs vectors and the use of both the x- and y-axes simultaneously.

The addition of another axis not only helps describe motion in two dimen-

sions but also simplifies analysis of motion in one dimension. For example,

two methods can be used to describe the motion of a jet moving at 300 m/s to

the northeast. In one approach, the coordinate system can be turned so that

the plane is depicted as moving along the y-axis, as in Figure 6(a). The jet’s

motion also can be depicted on a two-dimensional coordinate system whose

axes point north and east, as shown in Figure 6(b).One problem with the first method is that the axis must be turned again if

the direction of the plane changes. Another problem is that the first method

provides no way to deal with a second airplane that is not traveling in the

same direction as the first airplane. Thus, axes are often designated using fixed

directions. For example, in Figure 6(b), the positive y-axis points north and

the positive x-axis points east.

Similarly, when you analyze the motion of objects thrown into the air,

orienting the y-axis parallel to the vertical direction simplifies problem

solving.

Chapter 386

SECTION OBJECTIVES

■ Identify appropriate coordi-nate systems for solvingproblems with vectors.

■ Apply the Pythagorean theorem and tangent func-tion to calculate the magni-tude and direction of aresultant vector.

■ Resolve vectors into compo-nents using the sine andcosine functions.

■ Add vectors that are notperpendicular.

Teaching TipReview the sign conventions forcoordinate systems that wereestablished in the chapter“Motion in One Dimension.”Movements to the right along thex-axis and upward along the y-axis are considered positive, andmovements to the left along thex-axis and downward along they-axis are considered negative.

Visual Strategy

Figure 6Point out the two very differentchoices for coordinate axes.

Which set of axes will give thecorrect answer?

Either set of axes must givethe same results.A

Q

GENERAL

Δy

y

Figure 5A gecko’s displacement while climb-ing a tree can be represented by anarrow pointing along the y-axis.

v = 300 m/s northeast

yN

S

W E

(a)

Figure 6A plane traveling northeast at a velocity of 300 m/s can be repre-sented as either (a) moving along a y-axis chosen to point to thenortheast or (b) moving at an angle of 45° to both the x- and y-axes, which line up with west-east and south-north, respectively.

v = 300 m/s at 45°

y

(b)

x

Page 10: Compression Guide Two-Dimensional Motion and Vectorsaeascience.weebly.com/uploads/3/1/9/8/31980129/honors_vectors.pdf · Section 1 Introduction to Vectors ... SE Sample Set D Projectiles

87

SECTION 2

There are no firm rules for applying coordinate systems to situations involvingvectors. As long as you are consistent, the final answer will be correct regardless ofthe system you choose. Perhaps your best choice for orienting axes is the approachthat makes solving the problem easiest for you.

DETERMINING RESULTANT MAGNITUDE AND DIRECTION

In Section 1, the magnitude and direction of a resultant were found graphical-

ly. However, this approach is time consuming, and the accuracy of the answer

depends on how carefully the diagram is drawn and measured. A simpler

method uses the Pythagorean theorem and the tangent function.

Use the Pythagorean theorem to find the magnitude of the resultant

Imagine a tourist climbing a pyramid in Egypt. The tourist knows the height

and width of the pyramid and would like to know the distance covered in a

climb from the bottom to the top of the pyramid. Assume that the tourist

climbs directly up the middle of one face.

As can be seen in Figure 7, the magnitude of the tourist’s vertical displace-

ment, Δy, is the height of the pyramid. The magnitude of the horizontal dis-

placement, Δx, equals the distance from one edge of the pyramid to the

middle, or half the pyramid’s width. Notice that these two vectors are perpen-

dicular and form a right triangle with the displacement, d.As shown in Figure 8(a), the Pythagorean theorem states that for any right

triangle, the square of the hypotenuse—the side opposite the right angle—

equals the sum of the squares of the other two sides, or legs.

In Figure 8(b), the Pythagorean theorem is applied to find the tourist’s dis-

placement. The square of the displacement is equal to the sum of the square

of the horizontal displacement and the square of the vertical displacement. In

this way, you can find out the magnitude of the displacement, d.

PYTHAGOREAN THEOREM FOR RIGHT TRIANGLES

c2 = a2 + b2

(length of hypotenuse)2 = (length of one leg)2 + (length of other leg)2

MisconceptionAlert

Students often try to apply the Pythagorean theorem to tri-angles that do not contain a right angle. Point out that thePythagorean theorem can beused only with a right triangle.Some students may know theLaw of Cosines, which applies toall triangles. This law states thatc2 = a2 + b2 − 2ab cosq. The Lawof Cosines can be used to calcu-late one side of any triangle whenthe opposite angle and thelengths of the other two sides areknown. In this expression, c is theunknown side, q is the angleopposite c, and a and b are thetwo known sides. Some studentsmay attempt to use the Law ofCosines to add nonperpendicularvectors. This approach will givethe correct answer, but it entailsmore computation and is moreprone to student error whenmore than two vectors are to beadded.

Teaching TipPoint out that finding the resul-tant for the pyramid is fairly simple because the height, half-width, and hypotenuse form aright triangle. It is important tomention at this point that righttriangles will also allow studentsto find the x and y componentsthat are important for vectoraddition.

GENERAL

STOP

87Two-Dimensional Motion and Vectors

d

Δy

Δx

2Δx

Figure 7Because the base and height of apyramid are perpendicular, we canfind a tourist’s total displacement,d, if we know the height, Δy, andwidth, 2Δx, of the pyramid.

Δx

Δydc

c2 = a2 + b2 d2 = Δx2 + Δy2b

a

(a) (b)

Figure 8(a) The Pythagorean theoremcan be applied to any right tri-angle. (b) It can also be appliedto find the magnitude of a resul-tant displacement.

Page 11: Compression Guide Two-Dimensional Motion and Vectorsaeascience.weebly.com/uploads/3/1/9/8/31980129/honors_vectors.pdf · Section 1 Introduction to Vectors ... SE Sample Set D Projectiles

88

SECTION 2Use the tangent function to find the direction of the resultant

In order to completely describe the tourist’s displacement, you must also know

the direction of the tourist’s motion. Because Δx, Δy, and d form a right trian-

gle, as shown in Figure 9(b), the inverse tangent function can be used to find

the angle q, which denotes the direction of the tourist’s displacement.

For any right triangle, the tangent of an angle is defined as the ratio of the

opposite and adjacent legs with respect to a specified acute angle of a right tri-

angle, as shown in Figure 9(a).As shown below, the magnitude of the opposite leg divided by the magni-

tude of the adjacent leg equals the tangent of the angle.

The inverse of the tangent function, which is shown below, gives the angle.

q = tan−1�⎯o

a

p

d

p

j⎯�

DEFINITION OF THE TANGENT FUNCTION FOR RIGHT TRIANGLES

tan q = ⎯o

a

p

d

p

j⎯ tangent of angle = ⎯

o

ad

p

j

p

a

o

c

s

e

i

n

te

t l

l

e

e

g

g⎯

Chapter 388

Finding Resultant Magni-tude and DirectionA plane travels from Houston,Texas, to Washington, D.C., whichis 1540 km east and 1160 kmnorth of Houston. What is thetotal displacement of the plane?

Answer1930 km at 37.0° north of east

A camper travels 4.5 km northeastand 4.5 km northwest. What is thecamper’s total displacement?

Answer6.4 km north

SAMPLE PROBLEM A

Finding Resultant Magnitude and Direction

P R O B L E MAn archaeologist climbs the Great Pyramid in Giza,Egypt. The pyramid’s height is 136 m and its width is2.30 � 102 m. What is the magnitude and the directionof the displacement of the archaeologist after she hasclimbed from the bottom of the pyramid to the top?

S O L U T I O NGiven: Δy = 136 m Δx = ⎯1

2⎯(width) = 115 m

Unknown: d = ? q = ?

Diagram: Choose the archaeologist’s starting

position as the origin of the coordinate system.

Choose an equation or situation:The Pythagorean theorem can be used to find the magnitude of the archaeol-

ogist’s displacement. The direction of the displacement can be found by

using the tangent function.

d2 = Δx2 + Δy2

tan q = ⎯ΔΔ

x

y⎯

1. DEFINE

2. PLAN

θ

HypotenuseOpposite

Adjacent(a)

(b)

θtan = opp

adj

θtan = ΔyΔx

θ tan−1= Δy

Δx( )

θ

Δx

Δyd

Figure 9(a) The tangent function can beapplied to any right triangle, and (b) it can also be used to find thedirection of a resultant displacement.

Δy =136 m

Δx = 115 m

d

x

y

θ

Teaching Physics 1j* toMastery Students know how toresolve two-dimensional vectors into their components and calculatethe magnitude and direction of avector from its components.Activity Give students directionsfor a treasure hunt with move-ments that are all perpendicular toone another (i.e. move front/backand right/left). Have a student fol-low the directions to find a “trea-sure”. Now have the class do theproblem by resolving the direc-tions into two components andthen calculate the resultant vector.Have a student use this to godirectly from the start position tothe treasure.

Focus on the Standards

Page 12: Compression Guide Two-Dimensional Motion and Vectorsaeascience.weebly.com/uploads/3/1/9/8/31980129/honors_vectors.pdf · Section 1 Introduction to Vectors ... SE Sample Set D Projectiles

89

SECTION 2

Alternative Problem-Solving ApproachThe angle may be calculatedusing any trigonometric func-tion, such as the following.

q = sin−1�⎯Δd

y⎯� = 49.8°

ANSWERS

Practice A1. a. 23 km

b. 17 km to the east

2. 45.6 m at 9.5° east of north

3. 15.7 m at 22° to the side ofdownfield

4. 1.8 m at 49° below the horizontal

89Two-Dimensional Motion and Vectors

Rearrange the equations to isolate the unknowns:

d = �Δ�x�2�+� Δ�y�2�

q = tan−1�⎯ΔΔ

x

y⎯�

Substitute the values into the equations and solve:

d = �(1�15� m�)2� +� (�13�6�m�)2�

q = tan−1�⎯113

1

6

5

m

m⎯�

Because d is the hypotenuse, the archaeologist’s displacement should be less

than the sum of the height and half of the width. The angle is expected to be

more than 45° because the height is greater than half of the width.

q = 49.8°

d = 178 m

3. CALCULATE

4. EVALUATE

PRACTICE A

Finding Resultant Magnitude and Direction

1. A truck driver is attempting to deliver some furniture. First, he travels

8 km east, and then he turns around and travels 3 km west. Finally, he

turns again and travels 12 km east to his destination.

a. What distance has the driver traveled?

b. What is the driver’s total displacement?

2. While following the directions on a treasure map, a pirate walks 45.0 m

north and then turns and walks 7.5 m east. What single straight-line

displacement could the pirate have taken to reach the treasure?

3. Emily passes a soccer ball 6.0 m directly across the field to Kara. Kara

then kicks the ball 14.5 m directly down the field to Luisa. What is the

ball’s total displacement as it travels between Emily and Luisa?

4. A hummingbird, 3.4 m above the ground, flies 1.2 m along a straight

path. Upon spotting a flower below, the hummingbird drops directly

downward 1.4 m to hover in front of the flower. What is the humming-

bird’s total displacement?

Be sure your calculator is set tocalculate angles measured indegrees. Some calculators have abutton labeled “DRG” that,when pressed, toggles betweendegrees, radians, and grads.

re- SE Sample, 1–4;sultant Ch. Rvw.

21–22, 23*PW 2, 4–5, 7*PB Sample, 1–5

com- PW Sample, 1, 3*,ponent 6*

PB 6–10

PROBLEM GUIDE A

Solving for:

Use this guide to assign problems.SE = Student Edition TextbookPW = Problem WorkbookPB = Problem Bank on the

One-Stop Planner (OSP)

*Challenging ProblemConsult the printed Solutions Manual or the OSP for detailed solutions.

Page 13: Compression Guide Two-Dimensional Motion and Vectorsaeascience.weebly.com/uploads/3/1/9/8/31980129/honors_vectors.pdf · Section 1 Introduction to Vectors ... SE Sample Set D Projectiles

90

SECTION 2RESOLVING VECTORS INTO COMPONENTS

In the pyramid example, the horizontal and vertical parts that add up to give

the tourist’s actual displacement are called The x component is

parallel to the x-axis. The y component is parallel to the y-axis. Any vector can

be completely described by a set of perpendicular components.

In this textbook, components of vectors are shown as outlined, open

arrows. Components have arrowheads to indicate their direction. Compo-

nents are scalars (numbers), but they are signed numbers, and the direction is

important to determine their sign in a particular coordinate system.

You can often describe an object’s motion more conveniently by breaking a

single vector into two components, or resolving the vector. Resolving a vector

allows you to analyze the motion in each direction.

This point may be illustrated by examining a scene on the set of a new action

movie. For this scene, a biplane travels at 95 km/h at an angle of 20° relative to

the ground. Attempting to film the plane from below, a camera team travels in a

truck that is directly beneath the plane at all times, as shown in Figure 10.To find the velocity that the truck must maintain to stay beneath the plane,

we must know the horizontal component of the plane’s velocity. Once more,

the key to solving the problem is to recognize that a right triangle can be

drawn using the plane’s velocity and its x and y components. The situation

can then be analyzed using trigonometry.

The sine and cosine functions are defined in terms of the lengths of the

sides of such right triangles. The sine of an angle is the ratio of the leg oppo-

site that angle to the hypotenuse.

In Figure 11, the leg opposite the 20° angle represents the y component, vy ,

which describes the vertical speed of the airplane. The hypotenuse, vplane, is

the resultant vector that describes the airplane’s total velocity.

The cosine of an angle is the ratio between the leg adjacent to that angle

and the hypotenuse.

In Figure 11, the adjacent leg represents the x component, vx, which

describes the airplane’s horizontal speed. This x component equals the speed

that the truck must maintain to stay beneath the plane. Thus, the truck must

maintain a speed of vx = (cos 20°)(95 km/h) = 90 km/h.

DEFINITION OF THE COSINE FUNCTION FOR RIGHT TRIANGLES

cos q = ⎯h

a

y

d

p

j⎯ cosine of an angle = ⎯

a

h

d

y

j

p

a

o

c

t

e

e

n

n

t

u

le

se

g⎯

DEFINITION OF THE SINE FUNCTION FOR RIGHT TRIANGLES

sin q = ⎯o

h

p

yp

p⎯ sine of an angle = ⎯

o

h

p

y

p

p

o

o

s

te

it

n

e

u

le

se

g⎯

components.Teaching TipIn mathematics, the componentsof a vector are called projections.The x component is the projectionof the vector along the x-axis, andthe y component is the projectionof the vector along the y-axis.

MisconceptionAlert

Because of the prominence ofangles measured from the x-axis,students may develop the mis-conception that the x componentof a vector is always calculatedusing the cosine function. Thismisconception may be correctedby using examples on the boardin which the angles are measuredfrom the y-axis.

Teaching TipStudents who need a refresher ontrigonometry should be directedto Appendix A, which includes amore detailed discussion of thesine, cosine, and tangent func-tions, as well as the Pythagoreantheorem.

GENERALSTOP

Chapter 390

20˚

vplane

v truck

Figure 10A truck carrying a film crew mustbe driven at the correct velocity toenable the crew to film the under-side of a biplane. The plane flies at95 km/h at an angle of 20° relativeto the ground.

vplane = 95 km/h

vx

vy20°

Figure 11To stay beneath the biplane, thetruck must be driven with a velocityequal to the x component (vx) ofthe biplane’s velocity.

components of a vector

the projections of a vector alongthe axes of a coordinate system

Page 14: Compression Guide Two-Dimensional Motion and Vectorsaeascience.weebly.com/uploads/3/1/9/8/31980129/honors_vectors.pdf · Section 1 Introduction to Vectors ... SE Sample Set D Projectiles

91

SECTION 2

91Two-Dimensional Motion and Vectors

SAMPLE PROBLEM B

Resolving Vectors

P R O B L E MFind the components of the velocity of a helicopter traveling 95 km/h atan angle of 35° to the ground.

S O L U T I O NGiven: v = 95 km/h q = 35°

Unknown: vx = ? vy = ?

Diagram: The most convenient coordinate system is one

with the x-axis directed along the ground and

the y-axis directed vertically.

Choose an equation or situation:Because the axes are perpendicular, the sine and cosine functions

can be used to find the components.

sin q = ⎯v

v

y⎯

cos q = ⎯v

v

x⎯

Rearrange the equations to isolate the unknowns:

vy = v sin qvx = v cos q

Substitute the values into the equations and solve:

vy = (95 km/h)(sin 35°)

vx = (95 km/h)(cos 35°)

Because the components of the velocity form a right triangle with the

helicopter’s actual velocity, the components must satisfy the Pythagorean

theorem.

v2 = vx2 + vy

2

(95)2 = (78)2 + (54)2

9025 ≈ 9000

The slight difference is due to rounding.

vx = 78 km/h

vy = 54 km/h

1. DEFINE

2. PLAN

3. CALCULATE

4. EVALUATE

x

y v = 95 km/h

vx

vy

35°

Don’t assume that the cosinefunction can always be used forthe x-component and the sinefunction can always be used forthe y-component. The correctchoice of function depends onwhere the given angle is located.Instead, always check to seewhich component is adjacentand which component is oppo-site to the given angle.

Resolving VectorsAn arrow is shot from a bow at anangle of 25° above the horizontalwith an initial speed of 45 m/s.Find the horizontal and verticalcomponents of the arrow’s initialvelocity.

Answer41 m/s, 19 m/s

The arrow strikes the target witha speed of 45 m/s at an angle of−25° with respect to the horizon-tal. Calculate the horizontal andvertical components of thearrow’s final velocity.

Answer41 m/s, −19 m/s

one SE 1–2com- PW Sample, 1, 3ponent PB 1–4

both SE Sample, 3–4;com- Ch. Rvw. 24–25ponents PW 2–8

PB Sample, 5–10

PROBLEM GUIDE B

Solving for:

Use this guide to assign problems.SE = Student Edition TextbookPW = Problem WorkbookPB = Problem Bank on the

One-Stop Planner (OSP)

*Challenging ProblemConsult the printed Solutions Manual or the OSP for detailed solutions.

Page 15: Compression Guide Two-Dimensional Motion and Vectorsaeascience.weebly.com/uploads/3/1/9/8/31980129/honors_vectors.pdf · Section 1 Introduction to Vectors ... SE Sample Set D Projectiles

92

SECTION 2

ADDING VECTORS THAT ARE NOTPERPENDICULAR

Until this point, the vector-addition problems concerned vectors that are per-

pendicular to one another. However, many objects move in one direction and

then turn at an angle before continuing their motion.

Suppose that a plane initially travels 5 km at an angle of 35° to the ground,

then climbs at only 10° relative to the ground for 22 km. How can you deter-

mine the magnitude and direction for the vector denoting the total displace-

ment of the plane?

Because the original displacement vectors do not form a right triangle, you

can not apply the tangent function or the Pythagorean theorem when adding

the original two vectors.

Determining the magnitude and the direction of the resultant can be

achieved by resolving each of the plane’s displacement vectors into its x and y

components. Then the components along each axis can be added together. As

shown in Figure 12, these sums will be the two perpendicular components of

the resultant, d. The resultant’s magnitude can then be found by using the

Pythagorean theorem, and its direction can be found by using the inverse tan-

gent function.

ANSWERS

Practice B1. 95 km/h

2. 44 km/h

3. 21 m/s, 5.7 m/s

4. 0 m, 5 m

Teaching TipProblems involving vectors that arenot perpendicular use both vectoraddition and vector resolution.Because they act as a nice summa-ry to the concepts of this section,you may want to do several exam-ples involving this type of prob-lem. These problems require amethodical approach to problemsolving, which should prove help-ful to students while studyingmore-difficult subjects, such asinclined-plane problems and equilibrium problems.

Teaching TipBe sure students distinguishbetween the components of d1and d2 in Figure 12. Ask studentsto draw each vector and its com-ponents separately. Then, havethem sum the components ineach direction and add thesummed components together tofind d. Repeat with additionalexamples to help prepare stu-dents for Sample Problem C onthe next page.

GENERAL

Chapter 392

PRACTICE B

Resolving Vectors

1. How fast must a truck travel to stay beneath an airplane that is moving

105 km/h at an angle of 25° to the ground?

2. What is the magnitude of the vertical component of the velocity of the

plane in item 1?

3. A truck drives up a hill with a 15° incline. If the truck has a constant

speed of 22 m/s, what are the horizontal and vertical components of the

truck’s velocity?

4. What are the horizontal and vertical components of a cat’s displacement

when the cat has climbed 5 m directly up a tree?

Δx2

d2

d1 d

Δx1

Δy1

Δy2 Figure 12Add the components of the original dis-placement vectors to find two componentsthat form a right triangle with the resul-tant vector.

Page 16: Compression Guide Two-Dimensional Motion and Vectorsaeascience.weebly.com/uploads/3/1/9/8/31980129/honors_vectors.pdf · Section 1 Introduction to Vectors ... SE Sample Set D Projectiles

93

SECTION 2

93Two-Dimensional Motion and Vectors

STRATEGY

SAMPLE PROBLEM C

Adding Vectors Algebraically

P R O B L E MA hiker walks 27.0 km from her base camp at 35° south of east. The nextday, she walks 41.0 km in a direction 65° north of east and discovers a for-est ranger’s tower. Find the magnitude and direction of her resultant dis-placement between the base camp and the tower.

S O L U T I O NSelect a coordinate system. Then sketch and label each vector.

Given: d1 = 27.0 km q1 = −35°d2 = 41.0 km q2 = 65°

Unknown: d = ? q = ?

q1 is negative, because clockwise angles from the positive x-axis are conventionally considered to be negative.

Find the x and y components of all vectors.Make a separate sketch of the displacements for each day. Use the

cosine and sine functions to find the displacement components.

cos q = ⎯Δd

x⎯ sin q = ⎯

Δd

y⎯

For day 1: Δx1 = d1 cos q1 = (27.0 km) [cos (−35°)] = 22 km(a)

Δy1 = d1 sin q1 = (27.0 km) [sin (−35°)] = −15 km

For day 2: Δx2 = d2 cos q2 = (41.0 km) (cos 65°) = 17 km(b)

Δy2 = d2 sin q2 = (41.0 km) (sin 65°) = 37 km

Find the x and y components of the total displacement.

Δxtot = Δx1 + Δx2 = 22 km + 17 km = 39 km

Δytot = Δy1 + Δy2 = −15 km + 37 km = 22 km

Use the Pythagorean theorem to find the magnitude of theresultant vector.

d2 = (Δxtot)2 + (Δytot)

2

d = �(Δ�xt�ot�)2� +� (�Δ�yt�ot�)2� = �(3�9�km�)2� +� (�22� k�m�)2� =

Use a suitable trigonometric function to find the angle.

q = tan−1�⎯ΔΔx

yt

t

o

o

t

t⎯� = tan−1�⎯232

9

k

k

m

m⎯� = 29° north of east

45 km

1.

2.

3.

4.

5.

Adding Vectors AlgebraicallyA camper walks 4.5 km at 45°north of east then 4.5 km duesouth. Find the camper’s totaldisplacement.

Answer3.4 km at 22° south of east

A plane flies 118 km at 15.0°south of east and then flies118 km at 35.0° west of north.Find the magnitude and direc-tion of the total displacement of the plane.

Answer81 km at 55° north of east

d1

d2

d

Ranger’s tower

Basecamp

x

y

2θ1θθ

d1 = 27.0 km

Δx1

Δy1

x1 = −35°θ

Δy2

Δx2x

y

2 = 65°θ

d 2 =

41.

0 km

(b)

(a)

vector SE Sample, 1–3, 4*;sum Ch. Rvw. 26, 53*

PW Sample, 1–5PB Sample, 1–10

PROBLEM GUIDE C

Solving for:

Use this guide to assign problems.SE = Student Edition TextbookPW = Problem WorkbookPB = Problem Bank on the

One-Stop Planner (OSP)

*Challenging ProblemConsult the printed Solutions Manual or the OSP for detailed solutions.

Page 17: Compression Guide Two-Dimensional Motion and Vectorsaeascience.weebly.com/uploads/3/1/9/8/31980129/honors_vectors.pdf · Section 1 Introduction to Vectors ... SE Sample Set D Projectiles

94

SECTION 2

Chapter 394

PRACTICE C

Adding Vectors Algebraically

1. A football player runs directly down the field for 35 m before turning to

the right at an angle of 25° from his original direction and running an

additional 15 m before getting tackled. What is the magnitude and direc-

tion of the runner’s total displacement?

2. A plane travels 2.5 km at an angle of 35° to the ground and then changes

direction and travels 5.2 km at an angle of 22° to the ground. What is the

magnitude and direction of the plane’s total displacement?

3. During a rodeo, a clown runs 8.0 m north, turns 55° north of east, and runs

3.5 m. Then, after waiting for the bull to come near, the clown turns due east

and runs 5.0 m to exit the arena. What is the clown’s total displacement?

4. An airplane flying parallel to the ground undergoes two consecutive dis-

placements. The first is 75 km 30.0° west of north, and the second is

155 km 60.0° east of north. What is the total displacement of the

airplane?

SECTION REVIEW

1. Identify a convenient coordinate system for analyzing each of the follow-

ing situations:

a. a dog walking along a sidewalk

b. an acrobat walking along a high wire

c. a submarine submerging at an angle of 30° to the horizontal

2. Find the magnitude and direction of the resultant velocity vector for the

following perpendicular velocities:

a. a fish swimming at 3.0 m/s relative to the water across a river that

moves at 5.0 m/s

b. a surfer traveling at 1.0 m/s relative to the water across a wave that is

traveling at 6.0 m/s

3. Find the vector components along the directions noted in parentheses.

a. a car displaced 45° north of east by 10.0 km (north and east)

b. a duck accelerating away from a hunter at 2.0 m/s2 at an angle of 35°to the ground (horizontal and vertical)

4. Critical Thinking Why do nonperpendicular vectors need to be

resolved into components before you can add the vectors together?

1. a. x-axis: forward and back-ward on sidewalky-axis: left and right onsidewalk

b. x-axis: forward and back-ward on ropey-axis: up and down

c. x-axis: horizontal at waterlevely-axis: up and down

2. a. 5.8 m/s at 59° downriver from its intended path

b. 6.1 m/s at 9.5° from the direction the wave is traveling

3. a. 7.07 km north, 7.07 kmeast

b. 1.6 m/s2 horizontal, 1.1 m/s2 vertical

4. because the Pythagorean theorem and the tangentfunction can be applied only to right triangles

SECTION REVIEWANSWERS

Interactive Problem-ANSWERSPractice C1. 49 m at 7.3° to the right of

downfield

2. 7.5 km at 26° above the horizontal

3. 13.0 m at 57° north of east

4. 171 km at 34° east of north

Page 18: Compression Guide Two-Dimensional Motion and Vectorsaeascience.weebly.com/uploads/3/1/9/8/31980129/honors_vectors.pdf · Section 1 Introduction to Vectors ... SE Sample Set D Projectiles

(b)

vy

vx

SECTION OBJECTIVES

■ Recognize examples of pro-jectile motion.

■ Describe the path of a pro-jectile as a parabola.

■ Resolve vectors into theircomponents and apply thekinematic equations to solveproblems involving projectilemotion.

95Two-Dimensional Motion and Vectors 95

SECTION 3General LevelProjectile Motion SECTION 3

Teaching TipOn the chalkboard, show exam-ples of vector components and thekinematic equations. Show thesimplification of the x-directionequations when the x componentof acceleration is zero.

Visual Strategy

Figure 13Tell students that the longjumper builds up speed in the xdirection and jumps, so there isalso a component of speed in the y direction.

Does the angle of takeoffmatter to the jumper? Con-

sider the difference between avery small angle (near 0°) and alarger angle (near 45°).

The angle matters because itaffects how long the jumper

stays off the ground and how farhe goes horizontally while in theair.

A

Q

GENERAL

TWO-DIMENSIONAL MOTION

In the last section, quantities such as displacement and velocity were shown to

be vectors that can be resolved into components. In this section, these compo-

nents will be used to understand and predict the motion of objects thrown

into the air.

Use of components avoids vector multiplication

How can you know the displacement, velocity, and acceleration of a ball at any

point in time during its flight? All of the kinematic equations could be rewrit-

ten in terms of vector quantities. However, when an object is propelled into

the air in a direction other than straight up or down, the velocity, acceleration,

and displacement of the object do not all point in the same direction. This

makes the vector forms of the equations difficult to solve.

One way to deal with these situations is to avoid using the complicated vec-

tor forms of the equations altogether. Instead, apply the technique of resolving

vectors into components. Then you can apply the simpler one-dimensional

forms of the equations for each component. Finally, you can recombine the

components to determine the resultant.

Components simplify projectile motion

When a long jumper approaches his jump, he runs along a straight line, which

can be called the x-axis. When he jumps, as shown in Figure 13, his velocity

has both horizontal and vertical components. Movement in this plane can be

depicted by using both the x- and y-axes.

Note that in Figure 14(b), a jumper’s velocity vector is resolved into its two

vector components. This way, the jumper’s motion can be analyzed using the

kinematic equations applied to one direction at a time.

Figure 14(a) A long jumper’s velocity whilesprinting along the runway can berepresented by a horizontal vector.(b) Once the jumper is airborne,the jumper’s velocity at any instantcan be described by the compo-nents of the velocity.

Figure 13When the long jumper is in the air,his velocity has both a horizontaland a vertical component.

(a)

v

Page 19: Compression Guide Two-Dimensional Motion and Vectorsaeascience.weebly.com/uploads/3/1/9/8/31980129/honors_vectors.pdf · Section 1 Introduction to Vectors ... SE Sample Set D Projectiles

Path with air resistance

Path without air resistance

(a)(b)

96

SECTION 3In this section, we will focus on the form of two-dimensional motion called

Objects that are thrown or launched into the air and are

subject to gravity are called projectiles. Some examples of projectiles are soft-

balls, footballs, and arrows when they are projected through the air. Even a long

jumper can be considered a projectile.

Projectiles follow parabolic trajectories

The path of a projectile is a curve called a parabola, as shown in Figure 15(a).Many people mistakenly believe that projectiles eventually fall straight down in

much the same way that a cartoon character does after running off a cliff. But if

an object has an initial horizontal velocity in any given time interval, there will

be horizontal motion throughout the flight of the projectile. Note that for the

purposes of samples and exercises in this book, the horizontal velocity of the projec-

tile will be considered constant. This velocity would not be constant if we

accounted for air resistance. With air resistance, a projectile slows down as it col-

lides with air particles, as shown in Figure 15(b).

Projectile motion is free fall with an initial horizontal velocity

To understand the motion a projectile undergoes, first examine Figure 16. The

red ball was dropped at the same instant the yellow ball was launched horizon-

tally. If air resistance is disregarded, both balls hit the ground at the same time.

By examining each ball’s position in relation to the horizontal lines and to

one another, we see that the two balls fall at the same rate. This may seem

impossible because one is given an initial velocity and the other begins from

rest. But if the motion is analyzed one component at a time, it makes sense.

First, consider the red ball that falls straight down. It has no motion in the

horizontal direction. In the vertical direction, it starts from rest (vy,i = 0 m/s)

and proceeds in free fall. Thus, the kinematic equations from the chapter

“Motion in One Dimension” can be applied to analyze the vertical motion of

the falling ball, as shown on the next page. Note that on Earth’s surface the

acceleration (ay) will equal −g (−9.81 m/s2) because the only vertical compo-

nent of acceleration is free-fall acceleration. Note also that Δy is negative.

projectile motion.

Teaching TipReview the general equation for aparabola (y = ax2 + bx + c), andillustrate a parabola on the boardfor clarity. You may wish to pointout that for low launches, such asthe example shown in Figure 15,the path of a projectile is nearlythe same as a circular arc. Theparabolic shape becomes moreapparent for higher launches. Thepath of the yellow ball shown inFigure 16 is clearly a parabolarather than a circular arc.

Projectiles Launched

Chapter 396

Figure 15(a) Without air resistance, the soccer ball would travel along aparabola. (b) With air resistance,the soccer ball would travel along ashorter path.

Figure 16This is a strobe photograph of twotable-tennis balls released at thesame time. Even though the yellowball is given an initial horizontal veloc-ity and the red ball is simply dropped,both balls fall at the same rate.

projectile motion

the curved path that an objectfollows when thrown, launched,or otherwise projected near thesurface of Earth

Demonstration

Air ResistancePurpose Show the effects of airresistance on the flight of a projectile.Materials rubber stopper, table-tennis ballProcedure Toss the stopper at anangle of 45°. Have students notethe trajectory. Sketch the parabol-ic path on the chalkboard. Throwthe table-tennis ball at an angle of45°. Have students note the tra-jectory. Sketch its path on thechalkboard. Have students com-pare the two trajectories. Studentsshould note that the path of thetable-tennis ball is not symmetri-cal but has a steeper descent,demonstrating the effect of airresistance.

GENERAL

Page 20: Compression Guide Two-Dimensional Motion and Vectorsaeascience.weebly.com/uploads/3/1/9/8/31980129/honors_vectors.pdf · Section 1 Introduction to Vectors ... SE Sample Set D Projectiles

97

SECTION 3

Now consider the components of motion of the yellow ball that is

launched in Figure 16. This ball undergoes the same horizontal displacement

during each time interval. This means that the ball’s horizontal velocity

remains constant (if air resistance is assumed to be negligible). Thus, when

the kinematic equations are used to analyze the horizontal motion of a pro-

jectile, the initial horizontal velocity is equal to the horizontal velocity

throughout the projectile’s flight. A projectile’s horizontal motion is described

by the following equation.

Next consider the initial motion of the launched yellow ball in Figure 16.Despite having an initial horizontal velocity, the launched ball has no initial

velocity in the vertical direction. Just like the red ball that falls straight down,

the launched yellow ball is in free fall. The vertical motion of the launched yel-

low ball is described by the same free-fall equations. In any time interval, the

launched ball undergoes the same vertical displacement as the ball that falls

straight down. For this reason, both balls reach the ground at the same time.

To find the velocity of a projectile at any point during its flight, find the

vector that has the known components. Specifically, use the Pythagorean the-

orem to find the magnitude of the velocity, and use the tangent function to

find the direction of the velocity.

HORIZONTAL MOTION OF A PROJECTILE

vx = vx,i = constant

Δx = vx Δt

VERTICAL MOTION OF A PROJECTILE THAT FALLS FROM REST

vy,f = ay Δt

vy,f2 = 2ay Δy

Δy = ⎯12

⎯ay(Δt)2

97Two-Dimensional Motion and Vectors

The greatest distance a regulation-size baseball has ever been thrownis 135.9 m, by Glen Gorbous in 1957.

Did you know?

www.scilinks.orgTopic: Projectile MotionCode: HF61223

SAFETY

Perform this experiment away from wallsand furniture that can be damaged.

Roll a ball off a table. At the instant therolling ball leaves the table, drop a secondball from the same height above the floor.Do the two balls hit the floor at the sametime? Try varying the speed at which you

roll the first ball off the table. Does vary-ing the speed affect whether the two ballsstrike the ground at the same time? Nextroll one of the balls down a slope. Dropthe other ball from the base of the slopeat the instant the first ball leaves the slope.Which of the balls hits the ground first inthis situation?

Projectile Motion

MATERIALS LIST

• 2 identical balls• slope or ramp

TEACHER’S NOTESDropping the second ball as thefirst leaves the table is tricky. Stu-dents should try this severaltimes in order to get the timingright. Holding the second balljust past the edge of the table andnear the path of the first ballworks well. You should point outthe limitations of this Quick Labbecause of human reaction time.

Page 21: Compression Guide Two-Dimensional Motion and Vectorsaeascience.weebly.com/uploads/3/1/9/8/31980129/honors_vectors.pdf · Section 1 Introduction to Vectors ... SE Sample Set D Projectiles

98

SECTION 3

Chapter 398

SAMPLE PROBLEM D

Projectiles Launched Horizontally

P R O B L E MThe Royal Gorge Bridge in Colorado rises 321 m above the Arkansas River.Suppose you kick a rock horizontally off the bridge. The magnitude of therock’s horizontal displacement is 45.0 m. Find the speed at which the rockwas kicked.

S O L U T I O N1. DEFINE

2. PLAN

3. CALCULATE

4. EVALUATE

vx SE Sample, 1–3;Ch. Rvw. 31–32, 51a

PW 6, 7*, 8*PB 9

�x SE 4; Ch. Rvw. 33PW Sample, 1–2PB 8, 10

�y SE Ch. Rvw. 51bPW 3–4, 5*PB Sample, 1–7

PROBLEM GUIDE D

Solving for:

Use this guide to assign problems.SE = Student Edition TextbookPW = Problem WorkbookPB = Problem Bank on the

One-Stop Planner (OSP)

*Challenging ProblemConsult the printed Solutions Manual or the OSP for detailed solutions.

The value for vx can be eitherpositive or negative because ofthe square root. Because thedirection was not asked for, usethe positive root.

Given: Δy = −321 m Δx = 45.0 m ay = −g = −9.81 m/s2

Unknown: vi = vx = ?

Diagram: The initial velocity vector of the

rock has only a horizontal compo-

nent. Choose the coordinate system

oriented so that the positive y direc-

tion points upward and the positive

x direction points to the right.

Choose an equation or situation:Because air resistance can be neglected, the rock’s

horizontal velocity remains constant.

Δx = vxΔt

Because there is no initial vertical velocity, the following equation applies.

Δy = ⎯12

⎯ay(Δt)2

Rearrange the equations to isolate the unknowns:Note that the time interval is the same for the vertical and horizontal dis-

placements, so the second equation can be rearranged to solve for Δt.

Δt =�⎯2

a

Δ�y

y⎯�

Next rearrange the first equation for vx, and substitute the above value

of Δt into the new equation.

vx = ⎯ΔΔ

x

t⎯ = ��⎯

2

a

Δ�y

y⎯��Δx

Substitute the values into the equation and solve:

vx = �� (45.0 m) =

To check your work, estimate the value of the time interval for Δx and solve

for Δy. If vx is about 5.5 m/s and Δx = 45 m, Δt ≈ 8 s. If you use an approxi-

mate value of 10 m/s2 for g, Δy ≈ −320 m, almost identical to the given value.

5.56 m/s−9.81 m/s2

⎯⎯(2)(−321 m)

vx

ay

45.0 m

–321 m

yx

Projectiles LaunchedHorizontallyPeople in movies often jumpfrom buildings into pools. If aperson jumps horizontally fromthe 10th floor (30.0 m) to a poolthat is 5.0 m away from the build-ing, with what initial speed mustthe person jump?

Answer2.0 m/s

Page 22: Compression Guide Two-Dimensional Motion and Vectorsaeascience.weebly.com/uploads/3/1/9/8/31980129/honors_vectors.pdf · Section 1 Introduction to Vectors ... SE Sample Set D Projectiles

99

SECTION 3

Use components to analyze objects launched at an angle

Let us examine a case in which a projectile is launched at an angle to the hori-

zontal, as shown in Figure 17. The projectile has an initial vertical component

of velocity as well as a horizontal component of velocity.

Suppose the initial velocity vector makes an angle q with the horizontal.

Again, to analyze the motion of such a projectile, you must resolve the initial

velocity vector into its components. The sine and cosine functions can be used

to find the horizontal and vertical components of the initial velocity.

vx,i = vi cos q and vy,i = vi sin q

We can substitute these values for vx,i and vy,i into the kinematic equations

to obtain a set of equations that can be used to analyze the motion of a pro-

jectile launched at an angle.

As we have seen, the velocity of a projectile launched at an angle to the

ground has both horizontal and vertical components. The vertical motion is

similar to that of an object that is thrown straight up with an initial velocity.

PROJECTILES LAUNCHED AT AN ANGLE

vx = vx,i = vi cos q = constant

Δx = (vi cos q)Δt

vy,f = vi sin q + ay Δt

vy,f2 = vi

2 (sin q)2 + 2ay Δy

Δy = (vi sin q)Δt + ⎯12

⎯ay(Δt)2

99Two-Dimensional Motion and Vectors

PRACTICE D

Projectiles Launched Horizontally

1. A baseball rolls off a 0.70 m high desk and strikes the floor 0.25 m away

from the base of the desk. How fast was the ball rolling?

2. A cat chases a mouse across a 1.0 m high table. The mouse steps out of

the way, and the cat slides off the table and strikes the floor 2.2 m from

the edge of the table. When the cat slid off the table, what was its speed?

3. A pelican flying along a horizontal path drops a fish from a height of

5.4 m. The fish travels 8.0 m horizontally before it hits the water below.

What is the pelican’s speed?

4. If the pelican in item 3 was traveling at the same speed but was only

2.7 m above the water, how far would the fish travel horizontally before

hitting the water below?

vivy,i

vx,i

θ

Figure 17An object is projected with an initialvelocity, vi, at an angle of q . Resolvethe initial velocity into its x and ycomponents. Then, the kinematicequations can be applied todescribe the motion of the projec-tile throughout its flight.

ANSWERS

Practice D1. 0.66 m/s

2. 4.9 m/s

3. 7.6 m/s

4. 5.6 m

Visual Strategy

Figure 17Point out to students that fromthe time immediately after firinguntil it hits the ground, the pro-jectile follows a parabolic path.Emphasize that the vector vi doesnot represent any part of thepath of the projectile but onlythe direction and magnitude ofits initial velocity.

What is the acceleration of aprojectile just before it hits

the ground?

−9.81 m/s2, the same as atany other time during the

flight

Teaching TipStudents often memorize equa-tions without taking the time tounderstand them. Discouragestudents from memorizing theequations shown on this page.Help them see how each of theseequations is a form of one of thekinematic equations they learnedin the chapter “Motion in OneDimension.” Explain that solvingprojectile problems is done byapplying the kinematic equationsseparately in each direction.

A

Q

GENERAL

Page 23: Compression Guide Two-Dimensional Motion and Vectorsaeascience.weebly.com/uploads/3/1/9/8/31980129/honors_vectors.pdf · Section 1 Introduction to Vectors ... SE Sample Set D Projectiles

100

SECTION 3

Chapter 3100

STRATEGY

SAMPLE PROBLEM E

Projectiles Launched at an Angle

P R O B L E M

S O L U T I O NSelect a coordinate system.The positive y-axis points up, and the

positive x-axis points along the ground

toward the pole. Because the dart leaves

the gun at a height of 1.00 m, the vertical

distance is 4.00 m.

Use the inverse tangent function to find the angle that the initial velocitymakes with the x-axis.

q = tan−1�⎯ΔΔ

x

y⎯� = tan−1�⎯41.

0

0

.

0

0

m

m⎯� = 21.8°

Choose a kinematic equation to solve for time.Rearrange the equation for motion along the x-axis to isolate the unknown,

Δt, which is the time the dart takes to travel the horizontal distance.

Δx = (vi cos q)Δt

Δt = ⎯vic

Δo

x

sq⎯ = = 0.215 s

Find out how far each object will fall during this time.Use the free-fall kinematic equation in both cases. For the banana, vi = 0. Thus:

Δyb = ⎯12

⎯ay(Δt)2 = ⎯12

⎯(−9.81 m/s2)(0.215 s)2 = −0.227 m

The dart has an initial vertical component of velocity equal to vi sinq, so:

Δyd = (vi sin q)Δt + ⎯12

⎯ay(Δt)2

Δyd = (50.0 m/s)(sin 21.8°)(0.215 s) + ⎯12

⎯(−9.81 m/s2)(0.215 s)2

Δyd = 3.99 m − 0.227 m = 3.76 m

Analyze the results.Find the final height of both the banana and the dart.

ybanana, f = yb,i + Δyb = 5.00 m + (−0.227 m) =

ydart, f = yd,i + Δyd = 1.00 m + 3.76 m =

The dart hits the banana. The slight difference is due to rounding.

4.76 m above the ground

4.77 m above the ground

10.0 m⎯⎯⎯(50.0 m/s)(cos 21.8°)

1.

2.

3.

4.

5.

A zookeeper finds an escaped monkey hanging from a light pole. Aiming hertranquilizer gun at the monkey, she kneels 10.0 m from the light pole, whichis 5.00 m high. The tip of her gun is 1.00 m above the ground. At the samemoment that the monkey drops a banana, the zookeeper shoots. If the darttravels at 50.0 m/s, will the dart hit the monkey, the banana, or neither one?

x

10.0 m

4.00 m

1.00 mθ

y

vi

�y/�x SE Sample, 1–3;Ch. Rvw. 34a,35–36, 55a, 56b,59–60, 62a, 62c

PW 4b, 5, 7–8PB 6

vi SE 4; Ch. Rvw.47a*, 48a, 49*,56a, 61

PW Sample, 1–3, 4aPB 8, 10

�t SE Ch. Rvw. 34b,47b*, 55b, 62b

PB 7, 9

vf SE Ch. Rvw. 47c*,48b

q PW 6PB Sample, 1–5

PROBLEM GUIDE E

Solving for:

Use this guide to assign problems.SE = Student Edition TextbookPW = Problem WorkbookPB = Problem Bank on the

One-Stop Planner (OSP)

*Challenging ProblemConsult the printed Solutions Manual or the OSP for detailed solutions.

Projectiles Launched at an AngleA golfer practices driving balls offa cliff and into the water below.The edge of the cliff is 15 m abovethe water. If the golf ball islaunched at 51 m/s at an angle of15°, how far does the ball travelhorizontally before hitting thewater? (See Appendix A for hintson solving quadratic equations.)

Answer1.7 × 102 m

Page 24: Compression Guide Two-Dimensional Motion and Vectorsaeascience.weebly.com/uploads/3/1/9/8/31980129/honors_vectors.pdf · Section 1 Introduction to Vectors ... SE Sample Set D Projectiles

101

SECTION 3

101Two-Dimensional Motion and Vectors

PRACTICE E

Projectiles Launched at an Angle

1. In a scene in an action movie, a stuntman jumps from the top of one

building to the top of another building 4.0 m away. After a running start,

he leaps at a velocity of 5.0 m/s at an angle of 15° with respect to the flat

roof. Will he make it to the other roof, which is 2.5 m lower than the

building he jumps from?

2. A golfer hits a golf ball at an angle of 25.0° to the ground. If the golf ball

covers a horizontal distance of 301.5 m, what is the ball’s maximum

height? (Hint: At the top of its flight, the ball’s vertical velocity compo-

nent will be zero.)

3. A baseball is thrown at an angle of 25° relative to the ground at a speed

of 23.0 m/s. If the ball was caught 42.0 m from the thrower, how long

was it in the air? How high above the thrower did the ball travel?

4. Salmon often jump waterfalls to reach their breeding grounds. One salmon

starts 2.00 m from a waterfall that is 0.55 m tall and jumps at an angle of

32.0°. What must be the salmon’s minimum speed to reach the waterfall?

SECTION REVIEW

1. Which of the following exhibit parabolic motion?

a. a flat rock skipping across the surface of a lake

b. a three-point shot in basketball

c. the space shuttle while orbiting Earth

d. a ball bouncing across a room

e. a life preserver dropped from a stationary helicopter

2. During a thunderstorm, a tornado lifts a car to a height of 125 m above

the ground. Increasing in strength, the tornado flings the car horizon-

tally with a speed of 90.0 m/s. How long does the

car take to reach the ground? How far horizontal-

ly does the car travel before hitting the ground?

3. Interpreting Graphics An Alaskan rescue

plane drops a package of emergency rations to a

stranded party of explorers, as illustrated in

Figure 18. The plane is traveling horizontally at

30.0 m/s at a height of 200.0 m above the ground.

a. What horizontal distance does the package fall before landing?

b. Find the velocity of the package just before it hits the ground.

SECTION REVIEWANSWERS

200.0 m

vplane = 30.0 m/s

Figure 18

ANSWERS

Practice E1. yes, Δy = −2.3 m

2. 35.1 m

3. 2.0 s; 4.8 m

4. 6.2 m/s

Teaching TipPoint out that there are differentways to solve comparison ques-tions such as item 1 in Practice E.In this case, one option is to solvefor Δy and then compare the cal-culated value of Δy with the verti-cal distance given (2.5 m).Another option is to solve for Δxand compare the calculated valueof Δx with the horizontal distancegiven (4.0 m).

1. a, b, d

2. 5.05 s; 454 m

3. a. 192 mb. 69.4 m/s at 64.4° below

the horizontal

Page 25: Compression Guide Two-Dimensional Motion and Vectorsaeascience.weebly.com/uploads/3/1/9/8/31980129/honors_vectors.pdf · Section 1 Introduction to Vectors ... SE Sample Set D Projectiles

102

Relative MotionSECTION 4Advanced Level SECTION 4

FRAMES OF REFERENCE

If you are moving at 80 km/h north and a car passes you going 90 km/h, to

you the faster car seems to be moving north at 10 km/h. Someone standing on

the side of the road would measure the velocity of the faster car as 90 km/h

toward the north. This simple example demonstrates that velocity measure-

ments depend on the frame of reference of the observer.

Velocity measurements differ in different frames of reference

Observers using different frames of reference may measure different displace-

ments or velocities for an object in motion. That is, two observers moving with

respect to each other would generally not agree on some features of the motion.

Consider a stunt dummy that is dropped from an airplane flying horizontal-

ly over Earth with a constant velocity. As shown in Figure 19(a), a passenger on

the airplane would describe the motion of the dummy as a straight line toward

Earth. An observer on the ground would view the trajectory of the dummy as

that of a projectile, as shown in Figure 19(b). Relative to the ground, the

dummy would have a vertical component of velocity (resulting from free-fall

acceleration and equal to the velocity measured by the observer in the airplane)

and a horizontal component of velocity given to it by the airplane’s motion. If

the airplane continued to move horizontally with the same velocity, the dummy

would enter the swimming pool directly beneath the airplane (assuming negli-

gible air resistance).

Chapter 3102

SECTION OBJECTIVES

■ Describe situations in termsof frame of reference.

■ Solve problems involving relative velocity.

Visual Strategy

Figure 19Tell students that this diagramshows what would happen ifthere were no air resistance.

How would the two diagramschange if air resistance were

included?

In the first diagram, theobject would appear to fall

down and backward as viewedby the pilot. In the second dia-gram, the object would follow a shortened path as viewed bythe observer on the ground.(At any instant after release,both the x-component and they-component would be smaller,but the suppression of horizontalmotion has the dominant effecton the trajectory.)

A

Q

(a)

(b)

Figure 19When viewed from the plane (a), the stunt dummy (representedby the maroon dot) falls straight down. When viewed from a sta-tionary position on the ground (b), the stunt dummy follows aparabolic projectile path.

Page 26: Compression Guide Two-Dimensional Motion and Vectorsaeascience.weebly.com/uploads/3/1/9/8/31980129/honors_vectors.pdf · Section 1 Introduction to Vectors ... SE Sample Set D Projectiles

1. Elevator Acceleration A boy bounces arubber ball in an elevator that is going down. If theboy drops the ball as the elevator is slowing down, isthe magnitude of the ball’s acceleration relative to theelevator less than or greater than the magnitude ofits acceleration relative to the ground?

2.Aircraft Carrier Is thevelocity of a planerelative to an aircraftcarrier slower whenit approaches from the stern(rear) or from the bow (front)?

103

SECTION 4RELATIVE VELOCITY

The case of the faster car overtaking your car was easy to solve with a mini-

mum of thought and effort, but you will encounter many situations in which

a more systematic method of solving such problems is beneficial. To develop

this method, write down all the information that is given and that you want to

know in the form of velocities with subscripts appended.

vse = +80 km/h north (Here the subscript se means the velocity

of the slower car with respect to Earth.)

vfe = +90 km/h north (The subscript fe means the velocity

of the fast car with respect to Earth.)

We want to know vfs, which is the velocity of the fast car with respect to the

slower car. To find this, we write an equation for vfs in terms of the other

velocities, so on the right side of the equation the subscripts start with f and

eventually end with s. Also, each velocity subscript starts with the letter that

ended the preceding velocity subscript.

vfs = vfe + ves

The boldface notation indicates that velocity is a vector quantity. This

approach to adding and monitoring subscripts is similar to vector addition, in

which vector arrows are placed head to tail to find a resultant.

We know that ves = −vse because an observer in the slow car perceives

Earth as moving south at a velocity of 80 km/h while a stationary observer on

the ground (Earth) views the car as moving north at a velocity of 80 km/h.

Thus, this problem can be solved as follows:

vfs = vfe + ves = vfe − vse

vfs = (+90 km/h north) − (+80 km/h north) = +10 km/h north

When solving relative velocity problems, follow the above technique for

writing subscripts. The particular subscripts will vary depending on the prob-

lem, but the method for ordering the subscripts does not change. A general

form of the relative velocity equation is vac = vab + vbc. This general form may

help you remember the technique for writing subscripts.

Teaching TipRelative velocity can also beshown as the difference of twovectors.

vfs = vfe − vse

Another way of stating thisequation is that the relativevelocity of one moving object toanother is the difference betweentheir velocities relative to somecommon reference point.

You may want to demonstrateon the board that this equationworks for noncollinear velocities,as in Sample Problem F on thenext page.

ANSWERS

Conceptual Challenge1. greater than, because the ele-

vator is accelerating upwardtoward the ball as it falls

2. The plane’s velocity is slowerrelative to the moving carrierwhen it approaches from thestern.

103Two-Dimensional Motion and Vectors

Like velocity, displacement andacceleration depend on the frame inwhich they are measured. In somecases, it is instructive to visualizegravity as the ground acceleratingtoward a projectile rather than the projectile accelerating towardthe ground.

Did you know?

ADVANCED TOPICS

See “Special Relativity andVelocities” in Appendix J:Advanced Topics to learn abouthow velocities are added inEinstein’s special theory of relativity.

Why it Matters

Conceptual Challenge

Page 27: Compression Guide Two-Dimensional Motion and Vectorsaeascience.weebly.com/uploads/3/1/9/8/31980129/honors_vectors.pdf · Section 1 Introduction to Vectors ... SE Sample Set D Projectiles

104

SECTION 4

Chapter 3104

Relative VelocityA plane flies northeast at an air-speed of 563.0 km/h. (Airspeed isthe speed of an aircraft relative tothe air.) A 48.0 km/h wind isblowing to the southeast. What isthe plane’s velocity relative to theground?

Answer565.0 km/h at 40.1° north ofeast

The wind shifts to blow 63.0 km/htoward the southwest. What is theplane’s velocity relative to theground if the plane’s airspeedremains constant?

Answer500.0 km/h northeast

SAMPLE PROBLEM F

Relative Velocity

P R O B L E MA boat heading north crosses a wide river with a velocity of 10.00 km/hrelative to the water. The river has a uniform velocity of 5.00 km/h dueeast. Determine the boat’s velocity with respect to an observer on shore.

S O L U T I O NGiven: vbw = 10.00 km/h due north (velocity of the

boat, b, with respect to the water, w)

vwe = 5.00 km/h due east (velocity of the

water, w, with respect to Earth, e)

Unknown: vbe = ?

Diagram: See the diagram on the right.

Choose an equation or situation:To find vbe, write the equation so that the subscripts on the right start with b

and end with e.

vbe = vbw + vwe

As in Section 2, we use the Pythagorean theorem to calculate the magnitude of

the resultant velocity and the tangent function to find the direction.

(vbe)2 = (vbw)2 + (vwe)

2

tan q =

Rearrange the equations to isolate the unknowns:

vbe = �(vbw)2�+ (vwe)�2�

q = tan−1� �Substitute the known values into the equations and solve:

vbe = �(1�0.�00� k�m�/h�)2� +� (�5.�00� k�m�/h�)2�

q = tan−1� �

The boat travels at a speed of 11.18 km/h in the direction 26.6° east of north

with respect to Earth.

q = 26.6°

5.00 km/h⎯⎯10.0 km/h

vbe = 11.18 km/h

vwe⎯vbw

vwe⎯vbw

1. DEFINE

2. PLAN

3. CALCULATE

4. EVALUATE

vwe

vbw vbe

x

y

θN

S

W E

Teaching Physics 1h* toMastery Students know Newton’slaws are not exact but provide verygood approximations unless anobject is moving close to the speed of light or is small enough thatquantum effects are important.Activity Tell students that thereis one velocity that does not followthe rules for determining relativevelocity: the speed of light. Itsvelocity, 3.00 × 108 m/s, is constantin a vacuum. If an object, such as a spaceship, approached Earth at0.5 the speed of light, the lightreflected off the spaceship wouldnot approach Earth at 1.5 timesthe speed of light.

Focus on the Standards

Page 28: Compression Guide Two-Dimensional Motion and Vectorsaeascience.weebly.com/uploads/3/1/9/8/31980129/honors_vectors.pdf · Section 1 Introduction to Vectors ... SE Sample Set D Projectiles

105

SECTION 4

ANSWERS

Practice F1. 0 m/s

2. 14.5 m/s (in the direction thatthe aircraft carrier is moving)

3. 3.90 m/s at (4.0 × 101)°north of east

4. 26.4 m/s at 2.17° east of north

105Two-Dimensional Motion and Vectors

PRACTICE F

Relative Velocity

1. A passenger at the rear of a train traveling at 15 m/s relative to Earth

throws a baseball with a speed of 15 m/s in the direction opposite the

motion of the train. What is the velocity of the baseball relative to Earth

as it leaves the thrower’s hand?

2. A spy runs from the front to the back of an aircraft carrier at a velocity

of 3.5 m/s. If the aircraft carrier is moving forward at 18.0 m/s, how fast

does the spy appear to be running when viewed by an observer on a

nearby stationary submarine?

3. A ferry is crossing a river. If the ferry is headed due north with a speed of

2.5 m/s relative to the water and the river’s velocity is 3.0 m/s to the east,

what will the boat’s velocity relative to Earth be? (Hint: Remember to

include the direction in describing the velocity.)

4. A pet-store supply truck moves at 25.0 m/s north along a highway.

Inside, a dog moves at 1.75 m/s at an angle of 35.0° east of north.

What is the velocity of the dog relative to the road?

SECTION REVIEW

1. A woman on a 10-speed bicycle travels at 9 m/s relative to the ground as

she passes a little boy on a tricycle going in the opposite direction. If the

boy is traveling at 1 m/s relative to the ground, how fast does the boy

appear to be moving relative to the woman?

2. A girl at an airport rolls a ball north on a moving walkway that moves

east. If the ball’s speed with respect to the walkway is 0.15 m/s and the

walkway moves at a speed of 1.50 m/s, what is the velocity of the ball

relative to the ground?

3. Critical Thinking Describe the motion of the following objects

if they are observed from the stated frames of reference:

a. a person standing on a platform viewed from a train traveling north

b. a train traveling north viewed by a person standing on a platform

c. a ball dropped by a boy walking at a speed of 1 m/s viewed by the boy

d. a ball dropped by a boy walking 1 m/s as seen by a nearby viewer who

is stationary

v, q SE Sample, 1–4;Ch. Rvw. 43a,44a–b, 46a–b,57a–b

PW Sample, 1PB 7, 10

�t SE Ch. Rvw. 45,50a–b, 52, 54, 58

PW 2, 4, 6PB Sample, 1–5

�y/�x SE Ch. Rvw. 43bPW 5PB 6, 8–9

PROBLEM GUIDE F

Solving for:

Use this guide to assign problems.SE = Student Edition TextbookPW = Problem WorkbookPB = Problem Bank on the

One-Stop Planner (OSP)

*Challenging ProblemConsult the printed Solutions Manual or the OSP for detailed solutions.

1. 10 m/s (in the oppositedirection)

2. 1.51 m/s at 5.7° north of east

3. a. south with a speed equalto the train’s speed

b. moves northc. appears to fall straight

downd. moves in a parabola

SECTION REVIEWANSWERS

Page 29: Compression Guide Two-Dimensional Motion and Vectorsaeascience.weebly.com/uploads/3/1/9/8/31980129/honors_vectors.pdf · Section 1 Introduction to Vectors ... SE Sample Set D Projectiles

KinesiologyLisa Griffin was born in Toronto,Canada, and grew up in a smalltown in Ontario. Griffin says thatshe has always been interested in“the nature of reality and therelationship between the mindand the body.” As a high schoolstudent visiting her publiclibrary, she first learned that thelink between the mind and thebody was the central nervous sys-tem (CNS). As an undergraduateat the University of Guelph, shewas inspired by Dr. John Brooke,who taught that the spinal cord isalso part of the brain and has itsown forms of “thinking” andmemory.

CHAPTER 3

PHYSICS CAREERS

106106

Lisa Griffin applies an electrical stimulus to anerve in a patient’s wrist. This experiment testedthe best patterns of stimulation to recreatemovement in paralyzed hands.

Kinesiology

How does the body move? This question isjust one of the many that kinesiology con-tinually asks. To learn more about kinesiol-ogy as a career, read the interview withLisa Griffin, who teaches in the Depart-ment of Kinesiology and Health Educationat the University of Texas at Austin.

What training did you receive in order to become a kinesiologist?

I received a B.Sc. degree in human kinetics with aminor in biochemistry and M.Sc. and Ph.D. degrees inneuroscience. Kinesiology typically covers motorcontrol, biomechanics, and exercise physiology. Peo-ple who work in these branches are known as neu-roscientists, biomechanists, and physiologists,respectively.

What makes kinesiology interesting to you?The field of kinesiology allows me to explorehow the central nervous system (CNS) con-trols human movement. Thus we workwith people, and the findings of our workcan be used to help others.

What is the nature of yourresearch?

We record force output and singlemotor unit firing patterns from themuscles of human participants dur-ing fatigue and training.We thenuse these frequency patterns tostimulate their hands artificial-ly with electrical stimulation.We are working towarddeveloping an electricalstimulation system thatpeople with paralysiscould use to generatelimb movement. This couldhelp many who have spinalcord injuries from accidentsor brain damage from stroke.

How does your work address two-dimensionalmotion and vectors?

I investigate motor unit firing frequencies required togenerate force output from muscle over time. Thuswe record muscle contraction with strain gauge force

transducers, bridge amplifiers, an analog to digitalconverter, and a computer data acquisition andanalysis program. For example, the muscles ofthe thumb produce force in both x and y direc-tions.We record the x and y forces on two dif-ferent channels, and then we calculate theresultant force online so that we can view thenet output during contraction.

What are your most and leastfavorite things about your work?My favorite thing is coming up withnew ideas and working with studentswho are excited about their work.The thing I would most like tochange is the amount of time it takesto get the results of the experimentsafter you think of the ideas.

What advice would you offerto students who are interestedin this field?Do not underestimate the depth ofthe questions that can be addressedwith human participants.

Chapter 3106

PHYSICS

CAREERS

Page 30: Compression Guide Two-Dimensional Motion and Vectorsaeascience.weebly.com/uploads/3/1/9/8/31980129/honors_vectors.pdf · Section 1 Introduction to Vectors ... SE Sample Set D Projectiles

KEY IDEAS

Section 1 Introduction to Vectors• A scalar is a quantity completely specified by only a number with appropri-

ate units, whereas a vector is a quantity that has magnitude and direction.

• Vectors can be added graphically using the triangle method of addition, in

which the tail of one vector is placed at the head of the other. The resultant is

the vector drawn from the tail of the first vector to the head of the last vector.

Section 2 Vector Operations• The Pythagorean theorem and the inverse tangent function can be used to

find the magnitude and direction of a resultant vector.

• Any vector can be resolved into its component vectors by using the sine

and cosine functions.

Section 3 Projectile Motion• Neglecting air resistance, a projectile has a constant horizontal velocity

and a constant downward free-fall acceleration.

• In the absence of air resistance, projectiles follow a parabolic path.

Section 4 Relative Motion• If the frame of reference is denoted with subscripts (vab is the velocity of

object or frame a with respect to object or frame b), then the velocity of an

object with respect to a different frame of reference can be found by

adding the known velocities so that the subscript starts with the letter that

ends the preceding velocity subscript: vac = vab + vbc.• If the order of the subscripts is reversed, there is a change in sign; for exam-

ple, vcd = −vdc.

KEY TERMS

scalar (p. 82)

vector (p. 82)

resultant (p. 83)

components of a vector (p. 90)

projectile motion (p. 96)

107

CHAPTER 3

Highlights

Teaching TipAsk students to prepare a conceptmap for the chapter. The conceptmap should include most of thevocabulary terms, along withother integral terms or concepts.

HighlightsCHAPTER 3

107Two-Dimensional Motion and Vectors

Variable Symbols

Quantities Units

d (vector) displacement m meters

v (vector) velocity m/s meters/second

a (vector) acceleration m/s2 meters/second2

Δx (scalar) horizontal component m meters

Δy (scalar) vertical component m meters

Diagram Symbols

displacement vector

velocity vector

acceleration vector

resultant vector

component

PROBLEM SOLVING

See Appendix D: Equations for a summary of the equationsintroduced in this chapter. Ifyou need more problem-solvingpractice, see Appendix I: Additional Problems.

Page 31: Compression Guide Two-Dimensional Motion and Vectorsaeascience.weebly.com/uploads/3/1/9/8/31980129/honors_vectors.pdf · Section 1 Introduction to Vectors ... SE Sample Set D Projectiles

CHAPTER 3

108

ReviewReview

CHAPTER 3

ANSWERS1. A scalar represents the magni-

tude of a physical quantity.

2. no, must be equal and opposite

3. Speed is the magnitude ofvelocity.

4. 30 m/s east

5. no, because the scalar has nodirection

6. a. 5.00 units at 53.1° belowthe positive x-axis

b. 5.00 units at 53.1° abovethe positive x-axis

c. 8.54 units at 69.4° belowthe positive x-axis

d. 5.00 units at 127° clockwisefrom the positive x-axis

7. a. 5.20 m at 60.0° above thepositive x-axis

b. 3.00 m at 30.0° below thepositive x-axis

c. 3.00 m at 150° counter-clockwise from the positivex-axis

d. 5.20 m at 60.0° below thepositive x-axis

8. 7.9 m at 4.3° north of west

9. 15.3 m at 58.4° south of east

10. when the vectors point in thesame direction

11. a car moving straight andspeeding up, a car movingstraight and slowing down

12. 55; the maximum value of thevector sum is 80 units, theminimum value is 30 units

13. Yes, the distance from the tailof the first vector to the headof the last vector is zero.

VECTORS AND THE GRAPHICALMETHOD

Review Questions

1. The magnitude of a vector is a scalar. Explain thisstatement.

2. If two vectors have unequal magnitudes, can theirsum be zero? Explain.

3. What is the relationship between instantaneousspeed and instantaneous velocity?

4. What is another way of saying −30 m/s west?

5. Is it possible to add a vector quantity to a scalarquantity? Explain.

6. Vector A is 3.00 units in length and points along thepositive x-axis. Vector B is 4.00 units in length andpoints along the negative y-axis. Use graphicalmethods to find the magnitude and direction of thefollowing vectors:

a. A + Bb. A − Bc. A + 2Bd. B − A

7. Each of the displacement vectors A and B shown inthe figure below has a magnitude of 3.00 m. Graph-ically find the following:

a. A + Bb. A − Bc. B − Ad. A − 2B

x

y

3.00 m

3.00 m

30.0°

A

B

8. A dog searching for a bone walks 3.50 m south, then8.20 m at an angle of 30.0° north of east, and finally15.0 m west. Use graphical techniques to find thedog’s resultant displacement vector.

9. A man lost in a maze makes three consecutive dis-placements so that at the end of the walk he is backwhere he started, as shown below. The first displace-ment is 8.00 m westward, and the second is 13.0 mnorthward. Use the graphical method to find thethird displacement.

Conceptual Questions

10. If B is added to A, under what conditions does theresultant have the magnitude equal to A + B?

11. Give an example of a moving object that has a veloc-ity vector and an acceleration vector in the samedirection and an example of one that has velocityand acceleration vectors in opposite directions.

12. A student accurately uses the method for combin-ing vectors. The two vectors she combines havemagnitudes of 55 and 25 units. The answer that shegets is either 85, 20, or 55. Pick the correct answer,and explain why it is the only one of the three thatcan be correct.

13. If a set of vectors laid head to tail forms a closedpolygon, the resultant is zero. Is this statement true?Explain your reasoning.

X

Chapter 3108

Page 32: Compression Guide Two-Dimensional Motion and Vectorsaeascience.weebly.com/uploads/3/1/9/8/31980129/honors_vectors.pdf · Section 1 Introduction to Vectors ... SE Sample Set D Projectiles

109

3 REVIEW

14. Yes, the second componentcould be nonzero.

15. No, the hypotenuse is alwaysgreater than the legs.

16. Vector addition is combiningvectors to find a resultant vec-tor. Vector resolution is break-ing a vector into its componentvectors.

17. by resolving both vectors intotheir vector components,adding the correspondingcomponents together, andfinding the vector sum of thesummed components

18. They are equal and opposite.

19. if the vector is oriented at 45°from the axes

20. They form a closed trianglewhen laid head to tail.

21. a. 5 blocks at 53° north ofeast

b. 13 blocks

22. 42.7 yards

23. 61.8 m at 76.0° S of E (or S ofW), 25.0 m at 53.1° S of E (orS of W)

24. 108 m, −19.1 m

25. 2.81 km east, 1.31 km north

26. 2.40 × 102 m at 57.2° south ofwest

27. Both hit at the same time.

28. yes, neglecting air resistance

29. no, neglecting air resistance

30. The vertical components ofeach velocity vector will be thesame, but the thrown ball willalso have a horizontal compo-nent of velocity. As a result,the thrown ball will have agreater speed.

31. 45.1 m/s

VECTOR OPERATIONS

Review Questions

14. Can a vector have a component equal to zero andstill have a nonzero magnitude?

15. Can a vector have a component greater than itsmagnitude?

16. Explain the difference between vector addition andvector resolution.

17. How would you add two vectors that are not per-pendicular or parallel?

Conceptual Questions

18. If A + B equals 0, what can you say about the com-ponents of the two vectors?

19. Under what circumstances would a vector havecomponents that are equal in magnitude?

20. The vector sum of three vectors gives a resultantequal to zero. What can you say about the vectors?

Practice Problems

For problems 21–23, see Sample Problem A.

21. A girl delivering newspapers travels three blockswest, four blocks north, and then six blocks east.

a. What is her resultant displacement?b. What is the total distance she travels?

22. A quarterback takes the ball from the line of scrim-mage, runs backward for 10.0 yards, and then runssideways parallel to the line of scrimmage for15.0 yards. At this point, he throws a 50.0-yard for-ward pass straight down the field. What is the mag-nitude of the football’s resultant displacement?

23. A shopper pushes a cart 40.0 m south down oneaisle and then turns 90.0° and moves 15.0 m. Hethen makes another 90.0° turn and moves 20.0 m.Find the shopper’s total displacement. (There couldbe more than one correct answer.)

For problems 24–25, see Sample Problem B.

24. A submarine dives 110.0 m at an angle of 10.0° belowthe horizontal. What are the two components?

25. A person walks 25.0° north of east for 3.10 km.How far would another person walk due north anddue east to arrive at the same location?

For problem 26, see Sample Problem C.

26. A person walks the path shown below. The total tripconsists of four straight-line paths. At the end of thewalk, what is the person’s resultant displacementmeasured from the starting point?

PROJECTILE MOTION

Review Questions

27. A dart is fired horizontally from a dart gun, andanother dart is dropped simultaneously from thesame height. If air resistance can be neglected,which dart hits the ground first?

28. If a rock is dropped from the top of a sailboat’smast, will it hit the deck at the same point whetherthe boat is at rest or in motion at constant velocity?

29. Does a ball dropped out of the window of a movingcar take longer to reach the ground than onedropped at the same height from a car at rest?

30. A rock is dropped at the same instant that a ball atthe same elevation is thrown horizontally. Which willhave the greater speed when it reaches ground level?

Practice Problems

For problems 31–33, see Sample Problem D.

31. The fastest recorded pitch in Major League Baseballwas thrown by Nolan Ryan in 1974. If this pitchwere thrown horizontally, the ball would fall 0.809 m (2.65 ft) by the time it reached home plate,18.3 m (60 ft) away. How fast was Ryan’s pitch?

N

S

W E300.0 m

100.0 m

200.0 m

150.0 m30.0°60.0°

?

109Two-Dimensional Motion and Vectors

Page 33: Compression Guide Two-Dimensional Motion and Vectorsaeascience.weebly.com/uploads/3/1/9/8/31980129/honors_vectors.pdf · Section 1 Introduction to Vectors ... SE Sample Set D Projectiles

110

3 REVIEW

32. 3.3 s; 36 m/s

33. 11 m

34. a. 2.77 × 105 mb. 284 s

35. a. clears the goal by 1 mb. falling

36. 4.11 m

37. 80 m; 210 m

38. Displacement and velocitydepend on the frame of refer-ence in which they are mea-sured.

39. the coordinate system used todescribe the motion

40. Earth

41. a. 70 m/s eastb. 20 m/s

42. a. To the passenger, the ball appears to move in a straight line. To an outside observer, the ball moves along a parabolic trajectory.

b. The passenger would see the ball move backward,while the stationary ob-server would see no change from part (a).

32. A person standing atthe edge of a seasidecliff kicks a stone overthe edge with a speed of18 m/s. The cliff is 52 mabove the water’s sur-face, as shown at right.How long does it takefor the stone to fall tothe water? With whatspeed does it strike thewater?

33. A spy in a speed boat is being chased down a riverby government officials in a faster craft. Just as theofficials’ boat pulls up next to the spy’s boat, bothboats reach the edge of a 5.0 m waterfall. If the spy’sspeed is 15 m/s and the officials’ speed is 26 m/s,how far apart will the two vessels be when they landbelow the waterfall?

For problems 34–37, see Sample Problem E.

34. A shell is fired from the ground with an initial speedof 1.70 × 103 m/s (approximately five times thespeed of sound) at an initial angle of 55.0° to thehorizontal. Neglecting air resistance, find

a. the shell’s horizontal rangeb. the amount of time the shell is in motion

35. A place kicker must kick a football from a point36.0 m (about 40.0 yd) from the goal. As a result ofthe kick, the ball must clear the crossbar, which is3.05 m high. When kicked, the ball leaves theground with a speed of 20.0 m/s at an angle of 53°to the horizontal.

a. By how much does the ball clear or fall shortof clearing the crossbar?

b. Does the ball approach the crossbar while stillrising or while falling?

36. When a water gun is fired while being held horizon-tally at a height of 1.00 m above ground level, thewater travels a horizontal distance of 5.00 m. Achild, who is holding the same gun in a horizontalposition, is also sliding down a 45.0° incline at aconstant speed of 2.00 m/s. If the child fires the gunwhen it is 1.00 m above the ground and the watertakes 0.329 s to reach the ground, how far will thewater travel horizontally?

37. A ship maneuvers to within 2.50 × 103 m of anisland’s 1.80 × 103 m high mountain peak and fires aprojectile at an enemy ship 6.10 × 102 m on the otherside of the peak, as illustrated below. If the shipshoots the projectile with an initial velocity of 2.50 ×102 m/s at an angle of 75.0°, how close to the enemyship does the projectile land? How close (vertically)does the projectile come to the peak?

RELATIVE MOTION

Review Questions

38. Explain the statement “All motion is relative.”

39. What is a frame of reference?

40. When we describe motion, what is a common frameof reference?

41. A small airplane is flying at 50 m/s toward the east.A wind of 20 m/s toward the east suddenly beginsto blow and gives the plane a velocity of 70 m/s east.

a. Which vector is the resultant vector?b. What is the magnitude of the wind velocity?

42. A ball is thrown upward in the air by a passenger ona train that is moving with constant velocity.

a. Describe the path of the ball as seen by thepassenger. Describe the path as seen by a sta-tionary observer outside the train.

b. How would these observations change if thetrain were accelerating along the track?

Practice Problems

For problems 43–46, see Sample Problem F.

43. A river flows due east at 1.50 m/s. A boat crosses theriver from the south shore to the north shore bymaintaining a constant velocity of 10.0 m/s duenorth relative to the water.

a. What is the velocity of the boat as viewed byan observer on shore?

vi =1.80 × 103 m

75.0°

2.50 × 103 m

2.50 × 102 m/s

6.10 × 102 m

Chapter 3110

x

y

vi = +18 m/s

h = 52 mg

Page 34: Compression Guide Two-Dimensional Motion and Vectorsaeascience.weebly.com/uploads/3/1/9/8/31980129/honors_vectors.pdf · Section 1 Introduction to Vectors ... SE Sample Set D Projectiles

111

3 REVIEW

43. a. 10.1 m/s at 8.53° east ofnorth

b. 48.8 m

44. a. 14.1° north of westb. 199 km/h

45. 7.5 min

46. a. 23.2° upstream fromstraight across

b. 8.72 m/s across the river

47. a. 41.7 m/sb. 3.81 sc. vy,f = −13.5 m/s,

vx,f = 34.2 m/s,vf = 36.7 m/s

48. a. 15 m/sb. 15 m/s

49. 10.5 m/s

50. a. 22.2 sb. 2.00 × 102 s

51. a. 2.66 m/sb. 0.64 m

52. 22.5 s

53. 157 km

54. 7.0 × 101 s

b. If the river is 325 m wide, how far downstreamis the boat when it reaches the north shore?

44. The pilot of an aircraft wishes to fly due west in a50.0 km/h wind blowing toward the south. The speedof the aircraft in the absence of a wind is 205 km/h.

a. In what direction should the aircraft head?b. What should its speed relative to the ground be?

45. A hunter wishes to cross a river that is 1.5 km wideand that flows with a speed of 5.0 km/h. The hunteruses a small powerboat that moves at a maximumspeed of 12 km/h with respect to the water. What isthe minimum time necessary for crossing?

46. A swimmer can swim in still water at a speed of9.50 m/s. He intends to swim directly across a riverthat has a downstream current of 3.75 m/s.

a. What must the swimmer’s direction be?b. What is his velocity relative to the bank?

MIXED REVIEW47. A ball player hits a home run, and the baseball just

clears a wall 21.0 m high located 130.0 m fromhome plate. The ball is hit at an angle of 35.0° to thehorizontal, and air resistance is negligible. Assumethe ball is hit at a height of 1.0 m above the ground.

a. What is the initial speed of the ball?b. How much time does it take for the ball to

reach the wall?c. Find the components of the velocity and the

speed of the ball when it reaches the wall.

48. A daredevil jumps a canyon 12 m wide. To do so, hedrives a car up a 15° incline.

a. What minimum speed must he achieve toclear the canyon?

b. If the daredevil jumps at this minimumspeed, what will his speed be when he reachesthe other side?

49. A 2.00 m tall basketball player attempts a goal 10.00 m from the basket (3.05 m high). If he shootsthe ball at a 45.0° angle, at what initial speed musthe throw the basketball so that it goes through thehoop without striking the backboard?

50. An escalator is 20.0 m long. If a person stands onthe escalator, it takes 50.0 s to ride to the top.

a. If a person walks up the moving escalatorwith a speed of 0.500 m/s relative to the esca-lator, how long does it take the person to getto the top?

b. If a person walks down the “up” escalator withthe same relative speed as in item (a), howlong does it take to reach the bottom?

51. A ball is projected horizontally from the edge of atable that is 1.00 m high, and it strikes the floor at apoint 1.20 m from the base of the table.

a. What is the initial speed of the ball? b. How high is the ball above the floor when its

velocity vector makes a 45.0° angle with thehorizontal?

52. How long does it take an automobile traveling 60.0 km/h to become even with a car that is travel-ing in another lane at 40.0 km/h if the cars’ frontbumpers are initially 125 m apart?

53. The eye of a hurricane passes over Grand BahamaIsland. It is moving in a direction 60.0° north ofwest with a speed of 41.0 km/h. Exactly three hourslater, the course of the hurricane shifts due north,and its speed slows to 25.0 km/h, as shown below.How far from Grand Bahama is the hurricane 4.50 hafter it passes over the island?

54. A boat moves through a river at 7.5 m/s relative tothe water, regardless of the boat’s direction. If thewater in the river is flowing at 1.5 m/s, how longdoes it take the boat to make a round trip consistingof a 250 m displacement downstream followed by a250 m displacement upstream?

41.0 km/h

25.0 km/h

60.0°

N

S

EW

111Two-Dimensional Motion and Vectors

Page 35: Compression Guide Two-Dimensional Motion and Vectorsaeascience.weebly.com/uploads/3/1/9/8/31980129/honors_vectors.pdf · Section 1 Introduction to Vectors ... SE Sample Set D Projectiles

112

3 REVIEW

55. a. 32.5 mb. 1.78 s

56. a. 5.0 × 101 m/sb. 4.0 × 101 m

57. a. 57.7 km/h at 60.0° west ofthe vertical

b. 28.8 km/h straight down

58. 12.0 s

59. 18 m; 7.9 m

60. 15.3 m

61. 6.19 m/s downfield

62. a. 2.4 × 104 mb. 152 sc. 6.0 × 104 m

55. A car is parked on a cliff overlooking the ocean onan incline that makes an angle of 24.0° below thehorizontal. The negligent driver leaves the car inneutral, and the emergency brakes are defective. Thecar rolls from rest down the incline with a constantacceleration of 4.00 m/s2 and travels 50.0 m to theedge of the cliff. The cliff is 30.0 m above the ocean.

a. What is the car’s position relative to the baseof the cliff when the car lands in the ocean?

b. How long is the car in the air?

56. A golf ball with an initial angle of 34° lands exactly240 m down the range on a level course.

a. Neglecting air friction, what initial speedwould achieve this result?

b. Using the speed determined in item (a), findthe maximum height reached by the ball.

57. A car travels due east with a speed of 50.0 km/h.Rain is falling vertically with respect to Earth. Thetraces of the rain on the side windows of the carmake an angle of 60.0° with the vertical. Find thevelocity of the rain with respect to the following:

a. the carb. Earth

58. A shopper in a department store can walk up a sta-tionary (stalled) escalator in 30.0 s. If the normallyfunctioning escalator can carry the standing shop-per to the next floor in 20.0 s, how long would ittake the shopper to walk up the moving escalator?Assume the same walking effort for the shopperwhether the escalator is stalled or moving.

59. If a person can jump a horizontal distance of 3.0 mon Earth, how far could the person jump on themoon, where the free-fall acceleration is g/6 andg = 9.81 m/s2? How far could the person jump onMars, where the acceleration due to gravity is 0.38g?

60. A science student riding on a flatcar of a train mov-ing at a constant speed of 10.0 m/s throws a balltoward the caboose along a path that the studentjudges as making an initial angle of 60.0° with thehorizontal. The teacher, who is standing on theground nearby, observes the ball rising vertically.How high does the ball rise?

61. A football is thrown directly toward a receiver withan initial speed of 18.0 m/s at an angle of 35.0°above the horizontal. At that instant, the receiver is18.0 m from the quarterback. In what direction andwith what constant speed should the receiver run tocatch the football at the level at which it was thrown?

62. A rocket is launched at an angle of 53° above thehorizontal with an initial speed of 75 m/s, as shownbelow. It moves for 25 s along its initial line ofmotion with an acceleration of 25 m/s2. At thistime, its engines fail and the rocket proceeds tomove as a free body.

a. What is the rocket’s maximum altitude?b. What is the rocket’s total time of flight?c. What is the rocket’s horizontal range?

a = 25 m/s2

vi = 75 m/s

53°

Chapter 3112

Page 36: Compression Guide Two-Dimensional Motion and Vectorsaeascience.weebly.com/uploads/3/1/9/8/31980129/honors_vectors.pdf · Section 1 Introduction to Vectors ... SE Sample Set D Projectiles

113

3 REVIEW

113Two-Dimensional Motion and Vectors

1. Work in cooperative groups to analyze a game of

chess in terms of displacement vectors. Make a

model chessboard, and draw arrows showing all the

possible moves for each piece as vectors made of

horizontal and vertical components. Then have two

members of your group play the game while the

others keep track of each piece’s moves. Be prepared

to demonstrate how vector addition can be used to

explain where a piece would be after several moves.

2. Use a garden hose to investigate the laws of projec-

tile motion. Design experiments to investigate how

the angle of the hose affects the range of the water

stream. (Assume that the initial speed of water is

constant and is determined by the pressure indicat-

ed by the faucet’s setting.) What quantities will you

measure, and how will you measure them? What

variables do you need to control? What is the shape

of the water stream? How can you reach the maxi-

mum range? How can you reach the highest point?

Present your results to the rest of the class and dis-

cuss the conclusions.

3. You are helping NASA engineers design a basketball

court for a colony on the moon. How do you antici-

pate the ball’s motion compared with its motion on

Earth? What changes will there be for the players—

how they move and how they throw the ball? What

changes would you recommend for the size of the

court, the basket height, and other regulations in

order to adapt the sport to the moon’s low gravity?

Create a presentation or a report presenting your

suggestions, and include the physics concepts behind

your recommendations.

4. There is conflicting testimony in a court case. A

police officer claims that his radar monitor indicated

that a car was traveling at 176 km/h (110 mi/h). The

driver argues that the radar must have recorded the

relative velocity because he was only going 88 km/h

(55 mi/h). Is it possible that both are telling the

truth? Could one be lying? Prepare scripts for expert

witnesses, for both the prosecution and the defense,

that use physics to justify their positions before the

jury. Create visual aids to be used as evidence to sup-

port the different arguments.

Alternative Assessment

Alternative AssessmentANSWERS

1. Chess pieces can move inways that require more thanone component. Studentreports should show howseveral moves can be report-ed as a vector sum.

2. Student plans should be safeand include measurementsof angle and range. Theyshould find that 45° is thebest angle for maximumrange and 90° is the bestangle for maximum height.

3. Students should recognizethat balls will stay in the airlonger and that players canjump higher. Players willneed to shoot lower andmore gently, or the courtshould be longer and the basket higher.

4. Students should recognizethat if the cars were drivingtoward each other, bothcould be telling the truth.

Graphing Calculator PracticeVisit go.hrw.com for answers to thisGraphing Calculator activity.

Keyword HF6TDMXT

versus the time interval (X) that the ball remains in

the air. Why is the factor sin q missing from the

equation for Y1?

In this activity, you will determine the maximum

height and flight time of a baseball thrown vertically

at various initial velocities.

Visit go.hrw.com and type in the keyword

HF6TDMX to find this graphing calculator activity.

Refer to Appendix B for instructions on download-

ing the program for this activity.

Two Dimensional MotionRecall the following equation from your studies of

projectiles launched at an angle.

Δy = (vi sin q)Δt + ⎯12

⎯ay(Δt)2

Consider a baseball that is thrown straight up in the

air. The equation for projectile motion can be

entered as Y1 on a graphing calculator.

Y1 = VX − 4.9X2

Given the initial velocity (V), your graphing calcula-

tor can calculate the height (Y1) of the baseball

Page 37: Compression Guide Two-Dimensional Motion and Vectorsaeascience.weebly.com/uploads/3/1/9/8/31980129/honors_vectors.pdf · Section 1 Introduction to Vectors ... SE Sample Set D Projectiles

Standardized Test PrepCHAPTER 3

114

ANSWERS

1. B

2. H

3. A

4. J

5. C

6. G

7. B

8. J

MULTIPLE CHOICE

1. Vector A has a magnitude of 30 units. Vector B isperpendicular to vector A and has a magnitude of40 units. What would the magnitude of the resul-tant vector A + B be?

A. 10 unitsB. 50 unitsC. 70 unitsD. zero

2. What term represents the magnitude of a velocityvector?

F. accelerationG. momentumH. speedJ. velocity

Use the diagram below to answer questions 3–4.

3. What is the direction of the resultant vector A + B?

A. 15° above the x-axisB. 75° above the x-axisC. 15° below the x-axisD. 75° below the x-axis

4. What is the direction of the resultant vector A − B?

F. 15° above the x-axisG. 75° above the x-axisH. 15° below the x-axisJ. 75° below the x-axis

2.0 cm

30.0°2.0 cm

x

y

A

B

Use the passage below to answer questions 5–6.

A motorboat heads due east at 5.0 m/s across a riverthat flows toward the south at a speed of 5.0 m/s.

5. What is the resultant velocity relative to an observ-er on the shore?

A. 3.2 m/s to the southeastB. 5.0 m/s to the southeastC. 7.1 m/s to the southeastD. 10.0 m/s to the southeast

6. If the river is 125 m wide, how long does the boattake to cross the river?

F. 39 sG. 25 sH. 17 sJ. 12 s

7. The pilot of a plane measures an air velocity of165 km/h south relative to the plane. An observeron the ground sees the plane pass overhead at avelocity of 145 km/h toward the north. What is thevelocity of the wind that is affecting the plane rela-tive to the observer?

A. 20 km/h to the northB. 20 km/h to the southC. 165 km/h to the northD. 310 km/h to the south

8. A golfer takes two putts to sink his ball in the holeonce he is on the green. The first putt displaces theball 6.00 m east, and the second putt displaces theball 5.40 m south. What displacement would putthe ball in the hole in one putt?

F. 11.40 m southeastG. 8.07 m at 48.0° south of eastH. 3.32 m at 42.0° south of eastJ. 8.07 m at 42.0° south of east

Chapter 3114

Standardized Test Prep

Page 38: Compression Guide Two-Dimensional Motion and Vectorsaeascience.weebly.com/uploads/3/1/9/8/31980129/honors_vectors.pdf · Section 1 Introduction to Vectors ... SE Sample Set D Projectiles

115

9. D

10. H

11. B

12. H

13. They are perpendicular.

14. 31.5 m horizontally, 26.4 mvertically

15. 29.4 m/s

16. 10.8 m (See the SolutionsManual or the One-StopPlanner for a full solution.)

17. Executive C is correct. Stu-dent explanations shouldinclude the concept of rela-tive velocity—when a heli-copter or airship lifts offstraight up from the ground,it is already moving horizon-tally with Earth’s horizontalvelocity. (We assume thatEarth’s motion is constantfor the purposes of this sce-nario and does not dependon time.)

Use the information below to answer questions 9–12.

A girl riding a bicycle at 2.0 m/s throws a tennis ballhorizontally forward at a speed of 1.0 m/s from aheight of 1.5 m. At the same moment, a boy standingon the sidewalk drops a tennis ball straight down froma height of 1.5 m.

9. What is the initial speed of the girl’s ball relative tothe boy?

A. 1.0 m/sB. 1.5 m/sC. 2.0 m/sD. 3.0 m/s

10. If air resistance is disregarded, which ball will hitthe ground first?

F. the boy’s ballG. the girl’s ballH. neitherJ. The answer cannot be determined from the

given information.

11. If air resistance is disregarded, which ball will havea greater speed (relative to the ground) when ithits the ground?

A. the boy’s ballB. the girl’s ballC. neitherD. The answer cannot be determined from the

given information.

12. What is the speed of the girl’s ball when it hits theground?

F. 1.0 m/sG. 3.0 m/sH. 6.2 m/sJ. 8.4 m/s

SHORT RESPONSE

13. If one of the components of one vector along thedirection of another vector is zero, what can youconclude about these two vectors?

14. A roller coaster travels 41.1 m at an angle of 40.0°above the horizontal. How far does it move hori-zontally and vertically?

15. A ball is thrown straight upward and returns to thethrower’s hand after 3.00 s in the air. A second ballis thrown at an angle of 30.0° with the horizontal.At what speed must the second ball be thrown toreach the same height as the one thrown vertically?

EXTENDED RESPONSE

16. A human cannonball is shot out of a cannon at45.0° to the horizontal with an initial speed of 25.0m/s. A net is positioned at a horizontal distance of50.0 m from the cannon. At what height above thecannon should the net be placed in order to catchthe human cannonball? Show your work.

Read the following passage to answer question 17.

Three airline executives are discussing ideas for devel-oping flights that are more energy efficient.

Executive A: Because the Earth rotates from west toeast, we could operate “static flights”—a helicopter orairship could begin by rising straight up from NewYork City and then descend straight down four hourslater when San Francisco arrives below.

Executive B: This approach could work for one-wayflights, but the return trip would take 20 hours.

Executive C: That approach will never work. Thinkabout it. When you throw a ball straight up in the air, itcomes straight back down to the same point.

Executive A: The ball returns to the same point becauseEarth’s motion is not significant during such a shorttime.

17. In a paragraph, state which of the executives iscorrect, and explain why.

115Two-Dimensional Motion and Vectors

If you get stuck answering aquestion, move on. You can return to the questionlater if you have time.

Page 39: Compression Guide Two-Dimensional Motion and Vectorsaeascience.weebly.com/uploads/3/1/9/8/31980129/honors_vectors.pdf · Section 1 Introduction to Vectors ... SE Sample Set D Projectiles

Chapter 3116

When a ball rolls down an inclined plane, then rolls off the edge of a table, the

ball becomes a projectile with some positive horizontal velocity and an initial

vertical velocity of zero. However, the length of time that the projectile stays in

the air depends not on the horizontal velocity but on the height of the table

above the ground. The horizontal velocity determines how far the projectile

travels during the time it is in the air.

In this lab, you will roll a ball down an inclined plane, off the edge of a table,

and onto a piece of carbon paper on the floor. You will design your own experi-

ment by deciding the details of the setup and the procedure, including how many

trials to perform, the angles of the inclined plane, and how high up the plane you

will release the ball.Your experiment should include trials with the plane inclined

to different angles and multiple trials at different heights along the plane at each

angle of inclination. Your procedure should include steps to measure the height

from which the ball is released, the length of the ball’s travel along the plane, and

the horizontal displacement of the ball after it leaves the table.

PROCEDURE

1. Study the materials provided, and read the Analysis and Conclusions

questions. Design an experiment using the provided materials to meet

the goals stated above and to allow you to answer the questions.

2. Write out your lab procedure, including a detailed description of the

measurements to take during each step and the number of trials to per-

form. Create a data table to record your measurements for each trial. You

may use Figure 1 as a guide to one possible setup. Your setup should

include a box to catch the ball at the end of each trial.

3. Ask your teacher to approve your procedure.

4. Follow all steps of your procedure.

5. Clean up your work area as directed by your teacher.

OBJECTIVES

•Measure the velocity ofprojectiles in terms of thehorizontal displacementduring free fall.

•Compare the velocityand acceleration of pro-jectiles accelerated downdifferent inclined planes.

MATERIALS LIST• aluminum sheet, edges

covered with heavy tape• C-clamp• cardboard box• cord• inclined plane• masking tape• meterstick• packing tape• several large sheets white

paper• several sheets carbon paper• small metal ball• small metal washer• support stand and clamp• towel or cloth

116

Lab PlanningBeginning on page T34 arepreparation notes and teachingtips to assist you in planning.

Blank data tables (as well as some sample data) appear on the One-Stop Planner.

No Books in the Lab?See the Datasheets for In-Text Labs workbook for areproducible master copy ofthis experiment.

The same workbook also contains a version of this experi-ment with explicit proceduralsteps if you prefer a more di-rected approach.

Safety CautionRemind students to be careful;balls on the floor represent asafety hazard.

Tips and Tricks• With correct alignment, the box

should catch the ball everytime. Copy-paper boxes workwell.

• Carbon paper may be tapeddown along one side. Handlecarbon paper carefully and donot allow it to touch clothing.

• Show students how to use thewasher on a cord to find thebaseline from which to meas-ure the distance.

• Some students may need helpdetermining where to measurethe height and length of theinclined plane.

Inquiry LabCHAPTER 3Inquiry Lab

CHAPTER 3

Design Your Own

Velocity of a Projectile

SAFETY

• Tie back long hair, secure loose clothing, and remove loose jewelry toprevent its getting caught in moving or rotating parts. Put on goggles.

• Perform this experiment in a clear area. Falling or dropped massescan cause serious injury.

Page 40: Compression Guide Two-Dimensional Motion and Vectorsaeascience.weebly.com/uploads/3/1/9/8/31980129/honors_vectors.pdf · Section 1 Introduction to Vectors ... SE Sample Set D Projectiles

117

CHAPTER 3 LAB

ANSWERSAnalysis1. Δt = 0.39 s (for Δy = −0.75 m)

2. Trials 1 & 2: v = 2.22 m/sTrials 3 & 4: v = 1.54 m/sTrials 5 & 6: v = 1.64 m/sTrials 7 & 8: v = 1.17 m/s

3. Graphs should show a roughcorrelation between average hori-zontal velocity and height ofrelease.

4. Graphs should show no defi-nite correlation between averagehorizontal velocity and length oftravel along the plane.

Conclusions5. The greater the height ofrelease along the plane is, thegreater the average horizontalvelocity of the projectile is(because potential energy turnsinto kinetic energy as the ballrolls down the plane).

6. There is no definite correlationbetween length of travel along theplane and the average horizontalvelocity. At a given angle, longertravel along the plane gives agreater velocity because theheight of release is greater.

7. The height can be measuredwith more precision than thetime can be. Also, human reac-tion time introduces error.

8. The total velocity at impactwould be greatest in trials thathave the greatest average hori-zontal velocity.

Extension9. Plans should be complete andshould involve measuring timeintervals of projectiles launchedat different horizontal velocities.

117

ANALYSIS

1. Organizing Data Find the time interval for the

ball’s motion from the edge of the table to the floor

using the equation for the vertical motion of a projec-

tile. In those equations, Δy is the vertical displacement

of the ball after it leaves the table. The result is the time

interval for each trial.

2. Organizing Data Using the time interval from

item 1 and the value for Displacement Δx, calculate the

average horizontal velocity for each trial during the

ball’s motion from the edge of the table to the floor.

3. Constructing Graphs Plot a graph of average hori-

zontal velocity versus height of release. You may use

graph paper, a computer, or a graphing calculator.

4. Constructing Graphs Plot a graph of average hori-

zontal velocity versus length of travel along the plane.

You may use graph paper, a computer, or a graphing

calculator.

CONCLUSIONS

5. Drawing Conclusions What is the relationship

between the height of the inclined plane and the hori-

zontal velocity of the ball? Explain.

6. Drawing Conclusions What is the relationship

between the length of the inclined plane and the hori-

zontal velocity of the ball? Explain.

7. Evaluating Methods Why might using the vertical

displacement to calculate the time interval be more

reliable than using a stopwatch for each trial?

8. Applying Conclusions In which trials would the

total velocity of the ball when it hits the ground be the

greatest?

EXTENSION

9. Designing Experiments Design an experiment to

test the assumption that the time the ball is in the air is

independent of the horizontal velocity of the ball. If

you have time and your teacher approves your plan,

carry out the experiment.

Two-Dimensional Motion and Vectors

Figure 1

• Use tape to cover the sharp edges of the aluminumsheet before taping it to the end of the plane.The aluminum keeps the ball from bouncing as it rollsonto the table.

• Use a washer hanging from a string to find the zero-displacement point directly under the edge of the table.

• Use a box lined with a soft cloth to catch the ballafter it lands.

(a)

(b)